TS - Exam 1 Flashcards

1
Q

David and Kendall are going to the river to fish. They have previously formed a “fishing schemata” that provides them with the information needed about where to go, what boat to use, what to dress in, what bait to use, etc. They throw their baited fishing lines in the river and wait. A huge salmon takes the bait and puts up a terrific fight. Never having caught salmon before or any fish of this size, they are unprepared. David quickly thinks to grab hold of Kendall before he is pulled off the boat. Kendall quickly acts to lean backward and pull forcefully, while asking David to keep the boat stable. Finally, the huge salmon is successfully pulled from the water into the boat. David and Kendall are surprised and elated with their huge catch. According to Piaget, David and Kendall are:
Select one:

A.
In the process of cognitively organizing the new experience.

B.
Having difficulty incorporating the new knowledge because of their fishing schema.

C.
Adjusting their fishing schema through the process of adaptation.

D.
Assimilating and accommodating the new information in their schema.

A

The correct answer is C.

This is the most complete answer. David and Kendall are in the process of adjusting, enriching, and refining their fishing schema to accommodate the new experience. Choice A is true but not as complete as choice C. Organization and equilibration are further principles supporting adaptation; however, answer C goes one step further to explain what is really happening in the minds of David and Kendall. Choice B is incorrect, as this is unlikely to happen (on the other hand, a traumatic event might cause them to reject the experience). Choice D reflects the two main dynamics in the formulation of a schema, namely assimilation and accommodation, which are certainly occurring. However, the question describes the process of adaptation, which is the result of these two processes, resulting in adjustments to their schema as a function of the incorporation of the newly acquired knowledge. Hence, answer C is the best choice here.

How well did you know this?
1
Not at all
2
3
4
5
Perfectly
2
Q

According to the APA’s Ethical Standards (2002), psychologists should ensure that all information they distribute in public statements is:
Select one:

A.
Accurate and unbiased.

B.
Research-supported.

C.
Timely and informative.

D.
Documented and secure.

A

The language is very specific, and the correct answer is A.

Undoubtedly, timely and informative (answer C) are desirable qualities, but they are not the focus of the ethics code. Research-supported information and information stating security are not necessary for the psychologist to distribute in public, which makes answers B and D false.

How well did you know this?
1
Not at all
2
3
4
5
Perfectly
3
Q

__________ would be most likely to support the diathesis-stress model.
Select one:

A.
Skinner

B.
Watson

C.
Beck

D.
Pavlov

A

The correct answer is C.

Beck is best known for his cognitive therapy for depression, and the diathesis-stress model is a cognitive-behavioral construct. The diathesis-stress model hypothesizes that some people have genetic predispositions to acquire various mental or physical diseases (diathesis), but the chance that any disease will develop is increased by the kinds and levels of stress they are exposed to throughout life. In contrast to Beck, none of the other individuals prescribed to cognitive theories of behavior; in fact, Skinner, Watson, and Pavlov were strict behaviorists. Watson and Pavlov are best known for their work on classical conditioning (also known as Pavlovian conditioning), and Skinner is best known for his work on operant conditioning.

How well did you know this?
1
Not at all
2
3
4
5
Perfectly
4
Q

The tonic phase of a grand mal seizure is best characterized by:
Select one:

A.
Loss of consciousness.

B.
Rhythmic muscle spasms.

C.
Muscle rigidity.

D.
Automatism.

A

The correct answer is C.

The tonic phase of a grand mal seizure typically reveals muscular contraction and rigidity. Response A (loss of consciousness) follows the tonic-clonic phase. Response B (rhythmic muscular movements) marks the clonic phase. Response D (automatisms) are characteristic of complex partial seizures.

How well did you know this?
1
Not at all
2
3
4
5
Perfectly
5
Q

When in therapy with an individual, a multitheoretical psychotherapist would work interactively with:
Select one:

A.
Actions, systems, and behavior.

B.
Behavior, thoughts, and emotions.

C.
Feelings, motives, and actions.

D.
Thoughts, actions, and feelings.

A

The correct answer is D.

The multitheoretical psychotherapist works interactively with thoughts, feelings, and actions. Systems (answer A), behavior (answer B), and motives (answer C) are not the specific focus of multitheoretical psychotherapy.

How well did you know this?
1
Not at all
2
3
4
5
Perfectly
6
Q

In a study of college students, data on SAT scores, GPA, and scores on course finals are collected, as well as data including living situations, extracurricular activities, and social networks. Which statistical method can be used to collapse these variables into academic and social categories?
Select one:

A.
Factor analysis

B.
Discriminant analysis

C.
Analysis of variance

D.
Path analysis

A

The correct answer is A.

Factor analysis is used to group a larger number of variables into a smaller set of factors. Discriminant analysis, answer B, is used to predict group membership. ANOVA, answer C, is used to compare the means of two or more groups. Path analysis, answer D, is used to test the relationship between a number of IVs and a number of DVs simultaneously.

How well did you know this?
1
Not at all
2
3
4
5
Perfectly
7
Q

Which of the following is NOT a purpose of state licensing boards?
Select one:

A.
Ensure licensed psychologists have met minimal requirements for practicing psychology

B.
Regulate psychologists’ professional behavior based on the board’s ethics codes

C.
Monitor the conduct of licensed psychologists

D.
Protect the public

A

The correct answer is B.

It is not an accurate statement; though state boards regulate psychologists’ professional behavior, the regulation is based on laws, not ethics codes. It is true that state licensing boards protect the public (answer D) by ensuring that licensed psychologists have met minimal requirements for practicing psychology (answer A) and by monitoring the conduct of licensed psychologists (answer C).

How well did you know this?
1
Not at all
2
3
4
5
Perfectly
8
Q

If you were a psychologist working with a group of gay men who have been identified as high-risk for contracting HIV/AIDS, which of the following guidelines should you use when planning your intervention?
Select one:

A.
Discuss the pros and cons of unprotected sex

B.
Provide information about the dangers of HIV/AIDS and concrete steps to prevent infection

C.
Use a paradoxical intervention to convince them of the dangers of unprotected sex

D.
Rely on the power of group norms to help them avoid infection

A

The correct answer is B.

When planning an intervention it is best to attempt to provide useful information about HIV/AIDS, including the dangers and steps to prevent infection. Discussing the pros and cons (answer A) may be useful but not as effective as the description in answer B. Paradoxical intervention (answer C) is a technique where a clinician prescribes the problem and though it can be effective with certain situations, this would not be appropriate for this situation. Relying on group norms is not proactive enough by a psychologist to help avoid infection, so answer D is incorrect.

How well did you know this?
1
Not at all
2
3
4
5
Perfectly
9
Q

In cluster analysis, to form a second cluster, you first:
Select one:

A.
Find two variables that have a strong correlation with each other but a weak correlation with members of the first cluster.

B.
Find two variables that have a weak correlation with each other but a strong correlation with members of the first cluster.

C.
Find additional independent variables for the second cluster that were unrelated in the prior cluster so that there is no overlap between the two clusters.

D.
Use cluster sampling for the first cluster so that you can move on to the second cluster.

A

The correct answer is A.

Forming a second cluster in a cluster analysis begins with finding two strongly correlated variables that have weak correlations with items in the first cluster.

How well did you know this?
1
Not at all
2
3
4
5
Perfectly
10
Q

A higher verbal IQ score than performance IQ score on the WISC is suggestive of which diagnosis?
Select one:

A.
Agoraphobia

B.
Schizophrenia

C.
Depression

D.
OCD

A

The correct answer is D.

As anxiety is more detrimental to performance tasks as opposed to verbal tasks, which would create a higher verbal versus performance IQ. Responses A and B are incorrect; they are disorders not diagnosed in childhood. And C is incorrect, as depression likely would negatively impact both verbal and performance scores.

How well did you know this?
1
Not at all
2
3
4
5
Perfectly
11
Q

Taryn typically drinks a glass of warm milk at 10:00 p.m. to get ready for bed. After drinking the milk, she finds herself feeling warm and drowsy and so she continues to have a glass of milk every night as a way to get ready for bed. One night, she drinks milk at about 9:30 p.m. to try to sleep earlier. However, she notices she is not sleepy and continues to feel warm and drowsy at about 10:00 p.m. This is an example of:
Select one:

A.
Priming.

B.
Temporal conditioning.

C.
Delayed conditioning.

D.
Temporal delay.

A

The correct answer is B.

It is likely that Taryn has acquired a conditioned response (CR) of drowsiness to some other cues that regularly recur at 10:00 p.m. each day which are unrelated to the glass of milk. She was mistaken in her assumption that the milk was the stimulus that made her feel drowsy. Like priming (answer A), temporal conditioning is an implicit memory process because in both cases the individual is not consciously aware that recently encoded information is influencing his or her present behavior. But temporal conditioning is a more precise description of the particular implicit processes that seem to be operating in the present example because the relevant cues are related to time of day. Delayed conditioning (or delay conditioning) refers to a Pavlovian conditioning situation in which the beginning or onset of the conditioned stimulus (CS) precedes the onset of the unconditioned stimulus (US) but the two are present together for an overlapping period of time. This present example likely is an example of delayed conditioning (answer C) but it belongs to that more specific category of conditioning phenomena termed temporal conditioning because the operative CS are time-related cues. Temporal delay is a technical term for a pause, making answer D incorrect.

How well did you know this?
1
Not at all
2
3
4
5
Perfectly
12
Q

A man who dresses in women’s clothing for the purpose of sexual arousal is most likely to be diagnosed with:
Select one:

A.
Transsexualism.

B.
Gender dysphoria.

C.
Transvestic disorder with fetishism.

D.
Sexual masochism disorder.

A

The correct answer is C.

Men who dress in women’s clothing for the purpose of sexual arousal are defined as having transvestic disorder with fetishism. Answers A and B are both incorrect, as a transsexual person identifies with a gender not his/her own. Although gender dysphoria may be confused with transvestic disorder with fetishism, it is not a paraphilia. Answer D is incorrect; sexual masochism is a paraphilia, but entails to derive satisfaction from experiencing pain, humiliation, and suffering during sex.

How well did you know this?
1
Not at all
2
3
4
5
Perfectly
13
Q

Which of the following is NOT an APA Ethics Code General Principle?
Select one:

A.
Justice

B.
Fairness

C.
Integrity

D.
Fidelity

A

The correct answer is B.

Although fairness is similar to justice (answer A), it is not labeled as a General Principle in the Ethics Code. Justice, integrity, fidelity, and responsibility are all General Principles in the APA Ethics Code, which makes answers A, C, and D false.

How well did you know this?
1
Not at all
2
3
4
5
Perfectly
14
Q

Minuchin’s structural family therapy would most likely be concerned with:
Select one:

A.
Differentiation of the child.

B.
The father’s unemployment.

C.
The conscious mind.

D.
Family boundaries.

A

The correct answer is D.

Minuchin’s structural family therapy addresses boundaries (answer D), hierarchies, and coalitions. Differentiation (answer A) is addressed in Bowenian family systems therapy. Father’s unemployment (answer B) was identified as a source of family-wide distress by Nathan Ackerman. The conscious mind (answer C) is emphasized in Adlerian therapy, along with free will, striving for power, and a sense of inferiority.

How well did you know this?
1
Not at all
2
3
4
5
Perfectly
15
Q

Risk factors for developing somatic symptom disorder do NOT include:
Select one:

A.
Childhood illness.

B.
Low socioeconomic status.

C.
Personality trait of negative affectivity.

D.
Few years of formal education.

A

The correct answer is A.

Childhood illness has not been identified as a risk factor for somatic symptom disorder. Risk factors do include personality trait of negative affectivity, and environmental factors such as few years of formal education, low socioeconomic status, or recent experience of stressful events.

How well did you know this?
1
Not at all
2
3
4
5
Perfectly
16
Q

Random __________ involves participants in the population; random __________ involves participants in experimental conditions.
Select one:

A.
assignment; selection

B.
selection; assignment

C.
selection; selection

D.
assignment; assignment

A

The correct answer is B.

Random selection involves drawing participants from the population in such a way as to provide a representative sample; random assignment entails assigning participants to conditions in such a way that each participant has an equal probability of being assigned to any of the experimental conditions.

How well did you know this?
1
Not at all
2
3
4
5
Perfectly
17
Q

Which of the following factors is most associated with the development of shared psychotic disorder?
Select one:

A.
Social isolation

B.
Female gender

C.
Young age

D.
Low socioeconomic status

A

The correct answer is A.

Social isolation is often a precursor of shared psychotic disorder. Answers B, C, and D are incorrect, as they are not particular to this condition.

How well did you know this?
1
Not at all
2
3
4
5
Perfectly
18
Q

Which statement is true about Kubler-Ross’ theory of dying?
Select one:

A.
A person who cries and questions why he or she has to go through the grief is in the anger stage.

B.
A person who cries, yet attempts to distance him or herself from others, is in the denial stage.

C.
A person who cries, yet has acknowledged death, is in the depression stage.

D.
A person who cries and tries to settle unfinished business is in the acceptance stage.

A

The correct answer is A.

According to Kubler-Ross, the individual who asks questions like “why me?” is in the anger stage. A person who attempts to distance him or herself is in the depression stage, not the denial stage, which makes answer B false. A person who acknowledges their death is in the acceptance stage, not the depression stage, which makes answer C incorrect. An individual attempting to settle unfinished business is in the bargaining stage, which makes answer D incorrect.

How well did you know this?
1
Not at all
2
3
4
5
Perfectly
19
Q

In contrast to Kernberg’s conception of the etiology for narcissistic personality disorder, Kohut believed that:
Select one:

A.
The disorder results from a childhood in which caregivers were unresponsive and insensitive, leading to an unstable and weak self-representation.

B.
The disorder arises because infantile grandiosity was not challenged properly by caregivers; as such, the person was unable to achieve an integrated sense of self-representation and appropriate social standards.

C.
The disorder is the result of ego defenses, especially one of the projection of frustration and anger toward his/her caregiver onto others and his/her surroundings.

D.
The disorder is the result of an abnormal separation-individuation process due to the caregiver’s clinginess or minimal attention and involvement during childhood, leaving the person to cope with the fluctuation between fear of domination and fear of abandonment.

A

The correct answer is B.

As it accurately depicts Kohut’s views on a lack of developmental transition out of egocentrism and grandiosity. Answer A provides the classic views of Kernberg, a well-known source related to narcissism. Answer C deals with general defenses, and answer D is linked more with a Kleinian view of the development of narcissism, which makes these answers false.

How well did you know this?
1
Not at all
2
3
4
5
Perfectly
20
Q

__________ works around financial limitations set by insurance companies while trying to provide efficient insight and depth into a client’s life.
Select one:

A.
Brief psychodynamic therapy

B.
Gestalt therapy

C.
Existential therapy

D.
Narrative therapy

A

The correct answer is A.

Brief psychodynamic therapy (A) was designed to continue to provide insight and depth in a client’s life while meeting the limiting demands of insurance. Gestalt therapy (B) is focused on addressing the split-off parts of an individual and reintegrating the whole person. Existential therapy (C) is focused on the understanding and development of a personal meaning of life since, according to the philosophy, there is no inherent meaning. Narrative therapy (D) is focused on the fact that clients develop stories about themselves that are problem-saturated and filled with powerlessness. The purpose is then to re-story casting difficulty as a struggle for control.

How well did you know this?
1
Not at all
2
3
4
5
Perfectly
21
Q

When designing a study, it is possible to calculate the sample size that will be necessary to obtain a statistically significant effect. This calculation is called:
Select one:

A.
Type II error analysis.

B.
Effect size analysis.

C.
Power analysis.

D.
Type I error analysis.

A

The correct answer is C.

Although sample size calculations typically involve an estimate of expected effect size (answer B) and concern Type II error rates (answer A), the primary focus is actually on (1 - β), or power (i.e., the likelihood of finding an effect given that it does indeed exist). As such, this calculation is called a power analysis.

How well did you know this?
1
Not at all
2
3
4
5
Perfectly
22
Q

A manager who favors a total quality management (TQM) approach is most likely to:
Select one:

A.
Closely supervise and direct employee behavior.

B.
Involve employees and customers in decisions about quality improvement.

C.
Hire outside consultants to give advice on quality control.

D.
Add an additional level of managers so that employees can be more easily monitored.

A

The correct answer is B.

A TQM manager is most likely to involve employees and customers in decisions about quality improvement, as the focus of TQM emphasizes creativity and employee and customer involvement in both problem-solving and the elimination of flaws and errors at every level of the organization. TQM would not emphasize directing employee behavior (Answer A) or increasing monitoring of employees (Answer D), as one of the goals is increased autonomy and responsibility of the employees. TQM would not necessarily hire outside consultants (Answer C), as the goal is to use employee and customer input.

How well did you know this?
1
Not at all
2
3
4
5
Perfectly
23
Q

Centralization is appropriate when:
Select one:

A.
The environment is changeable and competitive.

B.
The environment is stable and predictable.

C.
The span of control is small.

D.
A manager possesses moderate authority.

A

The correct answer is B.

Centralized governance involves information management being reported up through a single chain of command. This structure of governance is utilized when the business environment is stable and predictable. Decentralized governance would be utilized when the business environment is considered to be changing and competitive (Answer A). Answers C and D are distractor terms.

How well did you know this?
1
Not at all
2
3
4
5
Perfectly
24
Q

Who developed multigenerational family therapy?
Select one:

A.
Nathan Ackerman

B.
Murray Bowen

C.
Carl Whitaker

D.
Salvador Minuchin

A

The correct answer is B.

Multigenerational family therapy is the premise of Murray Bowen’s family therapy, which involves a three-generation perspective, viewing the family as an emotional unit, with the goal of differentiation and individuation of family members. Failure of complementarity is a term from Nathan Ackerman’s (answer A) theory of family therapy from a psychoanalytic perspective; complementarity refers to harmony in social roles in a family system. Carl Whitaker (answer C) focused on an active and forceful role for the therapist, promoting flexibility in the family unit. He was not focused on techniques but concentrated on his own style, knowledge, and wisdom. Salvador Minuchin (answer D) developed structural family therapy, which involves a focus on hierarchies, boundaries, and alliances in the parent and child structure.

How well did you know this?
1
Not at all
2
3
4
5
Perfectly
25
Q

The earliest symptoms of major or mild neurocognitive disorder due to HIV infection are typically:
Select one:

A.
Delusions and social withdrawal.

B.
Apathy and muscle weakness.

C.
Forgetfulness and impaired concentration.

D.
Impaired problem-solving skills and agnosia.

A

The correct answer is C.

The earliest stages of major or mild neurocognitive disorder due to HIV infection include impaired attention/concentration and forgetfulness. The other responses are distractors, often appearing later in the disease progression.

How well did you know this?
1
Not at all
2
3
4
5
Perfectly
26
Q

Test items in the Developmental Test of Visual-Motor Integration consist of:
Select one:

A.
Various visual exercises.

B.
Various motor exercises.

C.
Geometric forms.

D.
Facial expressions that can be easily manipulated by the participants.

A

The correct answer is C.

The Developmental Test of Visual-Motor Integration consists of geometric forms of various complexities that the participant is required to copy in a test booklet. Items in A, B, and D are not used in this test.

How well did you know this?
1
Not at all
2
3
4
5
Perfectly
27
Q

Jennifer, who is 23 years old and a daily coffee drinker, explains that for about 7-10 days a month she usually experiences marked affective lability, irritability, and difficulty concentrating. Jennifer is likely to be diagnosed with:
Select one:

A.
caffeine withdrawal.

B.
premenstrual dysphoric disorder.

C.
persistent depressive disorder.

D.
hypersomnolence.

A

The correct answer is B.

The symptoms are episodic on a monthly cycle, and hence most likely related to hormonal fluctuations. Answer A is unlikely because Jennifer’s caffeine intake does not wax and wane over the month, so she is unlikely to be experiencing caffeine withdrawal. Persistent depressive disorder (answer C) does not fluctuate on a monthly cycle. No information relevant to hypersomnolence (answer D) was given in the question.

How well did you know this?
1
Not at all
2
3
4
5
Perfectly
28
Q

Typical antipsychotics such as Thorazine and haloperidol are less effective than atypical antipsychotics in:
Select one:

A.
Treating schizophrenia.

B.
Controlling positive symptoms of schizophrenia.

C.
Avoiding extrapyramidal side effects.

D.
Increasing dopamine levels.

A

The correct answer is C.

Typical antipsychotics are associated with higher rates of EPS. Answer A is too vague; typical antipsychotics are more effective in treating the positive symptoms of schizophrenia but quite ineffective in addressing the negative ones. Answer B is true, but an incorrect choice here. Answer D is incorrect, as antipsychotics, typical or atypical, decrease DA levels.

How well did you know this?
1
Not at all
2
3
4
5
Perfectly
29
Q

The Item Characteristic Curve provides all of the following information about a test item except:
Select one:

A.
Test item difficulty.

B.
Test item’s ability to discriminate between high and low scorers.

C.
Probability of answering the test item correctly by guessing.

D.
Degree to which the test item measures intended attribute.

A

The correct answer is D.

The Item Characteristic Curve provides information on difficulty, discrimination, and false positives. It does not provide information on the degree to which an item measures an attribute.

How well did you know this?
1
Not at all
2
3
4
5
Perfectly
30
Q

Arbitration can take a number of forms. When parties agree ahead of time to accept the recommendation of the arbitrator, they are engaging in:
Select one:

A.
voluntary arbitration.

B.
conventional arbitration.

C.
final offer arbitration.

D.
binding arbitration.

A

The correct answer is D.

Arbitration involves an authority figure that makes an agreement. Binding arbitration represents an acceptance of an agreement in advance, recommended by the arbitrator. Answer A is incorrect, as Voluntary arbitration represents the agreement to the arbitration process alone. Answer B is incorrect, as conventional arbitration has to do with the free choice of any settlement, while final offer arbitration, incorrect answer C, involves the selection of the last offer by the disputants.

How well did you know this?
1
Not at all
2
3
4
5
Perfectly
31
Q

Therapeutic techniques commonly employed in assertiveness training include:
Select one:

A.
Psychoeducation and cognitive restructuring.

B.
Relaxation exercises and systematic desensitization.

C.
Social skills groups and guided imagery.

D.
Behavior rehearsal and role-playing.

A

The correct answer is D.

Assertiveness training is used to help passive or aggressive individuals alter their verbal and nonverbal communication skills in such a way that their ability to communicate with others clearly and confidently is enhanced. The most commonly used techniques in assertiveness training are role-playing and behavior rehearsal. Answer A is incorrect because in cognitive restructuring, one attempts to replace an individual’s irrational, counterfactual thoughts with more accurate and beneficial beliefs. In contrast, assertiveness training focuses on the problematic behavior itself rather than the thoughts that may be behind those behaviors. Answers B and C are incorrect, as none of these techniques are typical components of assertiveness training.

How well did you know this?
1
Not at all
2
3
4
5
Perfectly
32
Q
Telemental health (TMH) can provide mental health services to people who may otherwise be unable to receive them. The delivery of TMH is favorable for a variety of clients and conditions, with the exception of those who:
Select one:

A.
Live in a remote area of the country.

B.
Are older or have a disability that causes limited mobility.

C.
Need specialized services due to a traumatic brain injury or a neurodegenerative condition.

D.
Are experiencing a crisis and require immediate intervention.

A

The correct answer is D.

Crises and life-threatening emergencies are not suitable for telemental health (TMH) treatment, as they require personal contact and assessment. Answers A, B, and C are good examples of situations in which TMH can be used effectively.

How well did you know this?
1
Not at all
2
3
4
5
Perfectly
33
Q

Factors that decrease the likelihood of undesirable message transmission include all of the following EXCEPT:
Select one:

A.
The transmitter of the message is unable to empathize with the recipient and experiences guilty feelings.

B.
The transmitter knows that the receiver of the news wants to hear the news.

C.
The intensity of the bad news is very high.

D.
The transmitter believes the recipient does not want to hear the news.

A

The correct answer is B.

The only response that would actually increase the likelihood of an undesirable message being transmitted is if the transmitter knows that the receiver of the news wants to hear the news. All other options are actually factors that decrease the likelihood of the transmission of undesirable messages.

How well did you know this?
1
Not at all
2
3
4
5
Perfectly
34
Q

Histrionic personality disorder is a diagnosis typically associated with women. Recent research suggests, however, that histrionic personality disorder is a sex-typed manifestation of another underlying condition. While women with this condition are likely to display histrionic traits, men are likely to manifest the condition through:
Select one:

A.
Dependent personality disorder.

B.
Schizoid personality disorder.

C.
Narcissistic personality disorder.

D.
Antisocial personality disorder.

A

The correct answer is D.

It may well be the case that, in men, the features of antisocial personality disorder are a male counterpart of the symptoms of histrionic personality disorder in women. None of the other responses have the corresponding features of emotional shallowness that make histrionic and antisocial personality disorders sex-typed manifestations of each other.

How well did you know this?
1
Not at all
2
3
4
5
Perfectly
35
Q

Damage to which region may result in difficulty coordinating muscle movements?
Select one:

A.
Secondary motor cortex

B.
Posterior motor cortex

C.
Motor association cortex

D.
Primary motor cortex

A

The correct answer is C.

Damage to the motor association cortex may result in difficulties coordinating muscle movement, such as the sequence of muscles required for speech.

How well did you know this?
1
Not at all
2
3
4
5
Perfectly
36
Q

The WISC-V test is appropriate for use through to what age?
Select one:

A.
13 years old

B.
15 years old

C.
16 years old

D.
17 years old

A

The correct answer is C.

The WISC-V is designed for use with individuals 16 years and under.

How well did you know this?
1
Not at all
2
3
4
5
Perfectly
37
Q

You are hired on as a group psychotherapist in a research study that follows strict manualized protocols dealing with anger management. The study calls for a comparison between rational emotive interventions and motivational interviewing. While working with the group that was randomly assigned to receive Rational Emotive Therapy in a group setting, you are most likely to do the following:
Select one:

A.
ask a group member to free-associate about his/her childhood.

B.
utilize stress inoculation with a group member.

C.
evaluate levels of differentiation.

D.
confront a group member about his or her irrational beliefs.

A

The correct answer is D.

A core therapeutic technique in Rational Emotive Therapy is the process of disputing irrational beliefs. This is commonly through the use of confrontation (D). Asking a group member to free-associate (A) is a psychodynamic technique, stress inoculation (B) is another therapy unrelated to REBT, and differentiation (C) is a component in Bowenian therapy.

How well did you know this?
1
Not at all
2
3
4
5
Perfectly
38
Q

Tom has lost the motivation to perform even the simplest tasks. He feels terrible and believes that bad things will always happen to him, things will never get better, and it is his own fault that things are so bad. Tom is experiencing:
Select one:

A.
Severe anxiety.

B.
Fundamental attribution error.

C.
Learned helplessness.

D.
Thoughts related to the cognitive triad.

A

The correct answer is C.

Learned helplessness is a condition in which an individual believes that he or she has no control over significant life circumstances. This often occurs when people perceive negative personal events in life as being constant, with no chance of improvement, regardless of what they do. Severe anxiety (answer A) is also referred to as a panic attack and is characterized by an episode of terror. The fundamental attribution error (answer B) is a tendency to overestimate the extent to which a person’s behavior is due to personality or dispositional characteristics rather than situational factors. The cognitive triad (answer D) is part of Beck’s cognitive theory of depression (self is worthless, the world is unfair, and the future is hopeless).

How well did you know this?
1
Not at all
2
3
4
5
Perfectly
39
Q

False memories are also referred to as:
Select one:

A.
Paramnesia.

B.
False consensus effect.

C.
Illusory perceptions.

D.
Amnesia.

A

The correct answer is A.

A false memory is a memory of an event that did not happen, or a distortion of an event that did occur. False memory is also referred to as paramnesia or illusory memory. The false consensus effect (answer B) is the tendency to overestimate the extent to which others agree with one’s ideas. Illusory perceptions or perceptual illusions (answer C) are not phenomena of memory but of perception. Amnesia (answer D) is a condition characterized by an impairment of long-term memory.

How well did you know this?
1
Not at all
2
3
4
5
Perfectly
40
Q

Job enrichment involves considering changes to all of the following aspects of a job EXCEPT:
Select one:

A.
Increased responsibility.

B.
Feedback on tasks.

C.
Task identity.

D.
Increase in salary.

A

The correct answer is D.

External factors, such as pay, are not considered part of job enrichment, as job enrichment is focused on higher-level needs of esteem and self-actualization (intrinsic factors). Answers A, B, and C are all areas that would be part of job enrichment, as the fulfillment of these is seen as being most motivating to employees.

How well did you know this?
1
Not at all
2
3
4
5
Perfectly
41
Q

Infantile amnesia is:
Select one:

A.
The lack of memory of a person’s initial three to four years of life.

B.
Usually caused by head injury.

C.
The lack of memory during a person’s initial six to seven years of life.

D.
Permanent.

A

The correct answer is A.

In the past, theorists assumed that children under the age of three had immature memory that would prevent them from being able to recall an event at a later time. This theory was based on the concept of infantile amnesia, which is an individual’s inability to recall information/events before three to four years of age. Implicit memory refers to the memory that is unconscious and helps an individual complete daily habits and motor tasks.

How well did you know this?
1
Not at all
2
3
4
5
Perfectly
42
Q

A clinician is aware of whether children in a group have been diagnosed with ADHD or not. As such, he tends to look for evidence to confirm ADHD among those children with a diagnosis and evidence to disconfirm ADHD among those children without a diagnosis. As such, when the clinician completes a measure of symptom ratings on each child, he inadvertently uses different standards to complete the measure depending on the child being assessed. If this measure of symptom ratings was compared to a newly designed measure of ADHD symptoms, there would be evidence of:
Select one:

A.
multicollinearity.

B.
composite criterion.

C.
criterion contamination.

D.
nonsystematic error.

A

The correct answer is C.

When systematic rater bias is introduced into a criterion (in this case, the measure of symptom ratings), it is evidence of criterion contamination. Composite criterion refers to multiple aspects being combined to create a criterion. Multicollinearity occurs when predictors are highly correlated. Nonsystematic error refers to random error; in the case of rater bias, there is systematic error.

How well did you know this?
1
Not at all
2
3
4
5
Perfectly
43
Q

Mary is a five-year-old girl suffering from nightly nightmares. The most effective CBT treatment for her is:
Select one:

A.
Coping skills training.

B.
Flooding.

C.
Stress inoculation.

D.
Stimulus discrimination.

A

The correct answer is A.

Conditions that cause chronic anxiety, such as nightmares, are best treated with coping skills training. Flooding (answer B) is an extinction approach that is rooted in behaviorism and avoidance of feared stimuli. Stress inoculation (answer C) is a four-phase training program for stress management often used in CBT. Stimulus discrimination (answer D) is the ability to distinguish between different stimuli.

How well did you know this?
1
Not at all
2
3
4
5
Perfectly
44
Q

The three subtypes of ADHD included in the DSM-5 are:
Select one:

A.
Predominantly inattentive presentation, predominantly hyperactive presentation, and predominantly impulsive presentation.

B.
Predominantly inattentive/oppositional presentation, predominantly hyperactive/impulsive presentation, and combined presentation.

C.
Predominantly inattentive presentation, predominantly hyperactive/impulsive presentation, and combined presentation.

D.
Predominantly impulsive presentation, primarily oppositional presentation, and predominantly hyperactive/inattentive presentation.

A

The correct answer is C.

Predominantly inattentive presentation, predominantly hyperactive/impulsive presentation, and combined presentation are the DSM-5 types of ADHD. The other answers include types of disorders that do not exist in the nomenclature, which makes answers A, B, and D false.

How well did you know this?
1
Not at all
2
3
4
5
Perfectly
45
Q

Choose the correct statement below:
Select one:

A.
When running multiple regression, the predictor with the lowest correlation with the criterion variable is given the most weight.

B.
An advantage of multiple regression over ANOVA is that multiple regression can use continuous or categorical data to measure independent variables (as opposed to only categorical data).

C.
When multiple regression is used in making predictions, high intercorrelations among the predictors and low correlations of each predictor with the criterion are needed.

D.
Predictors should intercorrelate.

A

The correct answer is B.

Although an ANOVA is limited to the use of categorical variables as independent variables, multiple regression may make use of both continuous variables and dummy-coded categorical data; in a multiple regression, the predictor most highly correlated with the dependent variable receives the most weight in the equation, and high intercorrelations between predictors (i.e., multicollinearity) result in unstable coefficient estimates (one wishes predictors to be highly correlated with the criterion variable).

How well did you know this?
1
Not at all
2
3
4
5
Perfectly
46
Q

In behavioral contrast, the relationship between the number of responses and the amount of reward is:
Select one:

A.
Linear.

B.
Inverse.

C.
Curvilinear.

D.
Tangential.

A

The correct answer is C.

It is not inverse (answer B) because in behavioral contrast, increases in reinforcement lead to increases in response rate and decreases in reinforcement lead to decreases in response rate. The relationship is not linear (answer A) because the response rate on a particular constant reinforcement schedule depends not only on the details of that schedule but also on the individual’s recent history of reinforcement. It is not tangential (answer D) but rather curvilinear (answer C), as the extent to which a given increase (or decrease) in reinforcement changes the individual’s response rate depends on his or her previous level of reinforcement.

How well did you know this?
1
Not at all
2
3
4
5
Perfectly
47
Q

Tanya is 30 years old and is hoping to soon get pregnant and start her family. Because she knows that she has a family history of Huntington’s disease (HD), she decided to talk with a genetic counselor. After gathering information regarding Tanya’s family history, the genetic counselor learns that Tanya’s father died of HD. What will the genetic counselor likely tell Tanya regarding the likelihood that her children will inherit HD?
Select one:

A.
Given that Tanya’s father had the disorder, the genetic counselor will advise Tanya that she has a 75% chance to pass the gene to her children, even if she has not inherited the gene herself.

B.
If a genetic test determines Tanya has HD, her children will have a 50% chance of inheriting the disease.

C.
HD is a sex-linked genetic disorder; therefore, the chance of inheritance depends on whether Tanya’s child is a boy or girl.

D.
If Tanya’s partner also has a family history of HD, their children will have a 25% chance of inheriting it.

A

The correct answer is B.

Huntington’s disease is an autosomal dominant genetic disorder, which means that if an individual inherits the gene for HD from one parent, the individual will have the disease. Answer A is incorrect, as Tanya will not pass the gene if she did not inherit it. In addition, she has a 50% (not 75%) chance to pass it down if she actually has it. To determine whether she inherited the disease from her father, she would have to undergo genetic testing to determine the number of trinucleotide repeats. If she has more than 35 repeats of CAG, then she inherited the gene for HD. The age of onset is typically between the mid-30s and mid-40s. Answer C is incorrect; because the disorder is autosomal, it is not sex-linked and therefore afflicts men and women equally. Answer D is also incorrect; if she has HD and her partner does not, her children will have a 50% chance of inheriting the disease. However, if both partners have the gene, they will pass it down to their children (100% heritability).

How well did you know this?
1
Not at all
2
3
4
5
Perfectly
48
Q

In a self-monitoring task, a person would be asked by a therapist to keep a record of thoughts related to a behavior, feelings related to a behavior, what occurred during a behavior, and __________ behavior.
Select one:

A.
time spent engaged in

B.
reinforcers related to a

C.
punishers related to a

D.
aversive outcomes related to

A

The correct answer is A.

In self-monitoring, the goal is to track the personal experience of a behavior, encompassing all related details of the behavior, including thoughts, feelings, and events during the behavior, and how much time was spent engaging (answer A) in the behavior. While reinforcers (answer B), punishers (answer C), and related aversive outcomes (also called punishers; answer D) may be identified through the process of collecting data through self-monitoring, they are not collected.

How well did you know this?
1
Not at all
2
3
4
5
Perfectly
49
Q

__________ can be a problem in individuals with strong fear or anxiety reactions to stimuli or situations that they encounter frequently and cannot completely predict, control, or avoid.
Select one:

A.
Counterconditioning

B.
Conditioned suppression

C.
Extinction

D.
Pseudoconditioning

A

The correct answer is B.

Conditioned suppression occurs when an individual temporarily suspends or significantly decreases the rate of an ongoing voluntary behavior (i.e., operant behavior), due to the occurrence of a stimulus that evokes a conditioned fear response. Conditioned suppression affects us all occasionally, but it can be particularly troublesome in individuals with strong fear or anxiety reactions, or phobias, to certain stimuli or situations that they encounter frequently and cannot completely predict, control, or avoid. Answer A is incorrect because counterconditioning is a Pavlovian conditioning technique that involves conditioning an individual to respond to a stimulus in a way that is incompatible with a response that has already been conditioned to that stimulus. Answer C is wrong because extinction is a decrease in either a conditioned response or operant behavior due to elimination of a reinforcer. Answer D is incorrect, because pseudoconditioning is a behavioral phenomenon in which an individual experiences a highly-salient US and, for a brief time thereafter, other stimuli are able to elicit a reflexive response that resembles the unconditioned response (UR).

How well did you know this?
1
Not at all
2
3
4
5
Perfectly
50
Q

The effects of malnutrition in children:
Select one:

A.
May be permanent.

B.
May be temporary.

C.
May be reversible if addressed early.

D.
May result in failure to thrive.

A

The correct answer is C.

Malnutrition is the state of having too few nutrients for the body to work properly. This condition is detrimental to people at all stages of life; however, those most impacted are infants still in the womb. Damage to the brain may be permanent if nutritional changes are not made early in a child’s life. Though both answers A and B could be correct, answer C is more specific. Failure to thrive is due to a lack of emotional support, not due to a lack of nutrition, making answer D incorrect.

How well did you know this?
1
Not at all
2
3
4
5
Perfectly
51
Q

A clinician is aware of whether children in a group have been diagnosed with ADHD or not. As such, he tends to look for evidence to confirm ADHD among those children with a diagnosis and evidence to disconfirm ADHD among those children without a diagnosis. As such, when the clinician completes a measure of symptom ratings on each child, he inadvertently uses different standards to complete the measure depending on the child being assessed. If this measure of symptom ratings was compared to a newly designed measure of ADHD symptoms, there would be evidence of:
Select one:

A.
Multicollinearity.

B.
Composite criterion.

C.
Criterion contamination.

D.
Nonsystematic error.

A

The correct answer is C.

When systematic rater bias is introduced into a criterion (in this case, the measure of symptom ratings), it is evidence of criterion contamination. Composite criterion (answer B) refers to multiple aspects being combined to create a criterion. Multicollinearity (answer A) occurs when predictors are highly correlated. Nonsystematic error (answer D) refers to random error; in the case of rater bias, there is systematic error.

How well did you know this?
1
Not at all
2
3
4
5
Perfectly
52
Q

Which of the following statements is true about temporal conditioning?
Select one:

A.
Temporal conditioning is a form of conditioning involving the brief presentation of a conditioning stimulus.

B.
Temporal conditioning is where the conditioned stimulus follows the unconditioned stimulus by a significant time period.

C.
Temporal conditioning is an operant conditioning term with no counterpart in classical conditioning.

D.
Temporal conditioning requires a constant time interval or a fixed interval schedule of reinforcement.

A

The correct answer is D.

Trace conditioning, not temporal conditioning, involves the conditioned stimulus following the unconditioned stimulus time period, which makes answer A incorrect. Delay conditioning involves the conditioned stimulus following the unconditioned stimulus by a significant time period, which makes answer B incorrect. Temporal conditioning is a classical conditioning term with no counterpart in operant conditioning, not the other way around, which makes answer C incorrect.

How well did you know this?
1
Not at all
2
3
4
5
Perfectly
53
Q

Dr. Perlow has seen Javier, an adolescent male client, for just two sessions when Javier reveals that he is gay. Dr. Perlow lacks any training in working with a GLBT population. Which of the following should be Dr. Perlow’s next step?
Select one:

A.
Quickly find current research articles that are relevant to gay males

B.
Refer Javier to another therapist who specializes in GLBT issues

C.
Begin the process of terminating therapy

D.
Seek consultation

A

The correct answer is D.

Dr. Perlow has questions about his professional competence with his case, which warrants consultation. Collecting a few research articles (answer A) will not likely contribute to competence in this area. Terminating (answer C) or referring the client (answer B) are potential options but are very problematic in that Javier is likely to interpret this as a rejection over an issue that is already heavily stigmatized by society.

How well did you know this?
1
Not at all
2
3
4
5
Perfectly
54
Q

A key principle underlying Montessori schools is that:
Select one:

A.
Maximum learning comes from individual instruction.

B.
Maximum learning comes from manipulation of materials.

C.
Maximum learning comes from small group instruction.

D.
Maximum learning comes from standardized curricula.

A

The correct answer is B.

Developed by Dr. Maria Montessori, the Montessori teaching method proposes that all learning is related to sensory perception, rather than through traditional classroom instruction and rote learning.

How well did you know this?
1
Not at all
2
3
4
5
Perfectly
55
Q

The adrenal medulla, which is located on the top of the kidneys, contains:
Select one:

A.
Testosterone.

B.
Aldosterone.

C.
Chromaffin cells.

D.
Neuroendocrine cells.

A

The correct answer is C.

Options A and B are both found in the adrenal cortex. Option D is a good answer, but chromaffin cells is a more accurate answer than neuroendocrine cells.

How well did you know this?
1
Not at all
2
3
4
5
Perfectly
56
Q

A pigeon has a high probability of eating and a low probability of pulling a lever. In order to increase the pigeon’s lever-pulling behavior, eating was made to be contingent on the pigeon pulling a lever. This is an example of:
Select one:

A.
Premack’s principle.

B.
Negative reinforcement.

C.
DRO.

D.
Shaping.

A

The correct answer is A.

Premack’s principle states that more-preferred behaviors can reinforce less-preferred behaviors. The relative preference for two behaviors can be determined by measuring the base rates at which the two behaviors are performed when the individual has the opportunity to do both. In such a free-choice situation, the behavior with the higher base rate is deemed as more preferred than the behavior with the lower base rate. Thus, a behavior with a higher base rate can be used to reinforce a behavior with a lower base rate. In the present example, eating is a high base-rate behavior, and lever-pulling is a low base-rate behavior, so eating can be used to reinforce lever-pulling. Negative reinforcement (answer B) is the procedure of removing an unpleasant stimulus or situation contingent on the performance of a particular behavior, with the aim of increasing that behavior. DRO (answer C) stands for differential reinforcement of other behavior, a behavior modification technique in which any behavior except the targeted inappropriate response is reinforced; typically, this results in a reduction of the inappropriate behavior. Answer D is incorrect, as shaping entails building a behavior by dividing it into small increments or steps and then teaching one step at a time until the desired behavior is achieved; this is not what is illustrated above. There is no “shaping” taking place, as the pigeon is rewarded in a predictable fashion (with food) every time he pulls the lever.

How well did you know this?
1
Not at all
2
3
4
5
Perfectly
57
Q

One theory that may explain stranger anxiety is:
Select one:

A.
Self-verification theory.

B.
Discrepancy theory.

C.
Synchrony theory.

D.
Attachment theory.

A

The correct answer is B.

Discrepancy refers to the idea that at around seven months of age, infants acquire schemes for familiar objects. When a new image or object is presented that differs from the old one, the child may experience uncertainty and anxiety. Synchrony (answer C) refers to the back-and-forth interactions between an infant and caregiver. Attachment theory (answer D) refers to the emotional bond that develops between an infant and his or her primary caregiver. Self-verification theory (answer A) refers to the idea that people are motivated to confirm their self-concept.

How well did you know this?
1
Not at all
2
3
4
5
Perfectly
58
Q

In schizoaffective disorder, the depressive type is characterized by all of the following EXCEPT:
Select one:

A.
More women than men have this type.

B.
The prognosis is better than for the bipolar type.

C.
The presence of major depressive episodes but not manic or mixed episodes.

D.
A later onset than for the bipolar type.

A

The correct answer is B.

The prognosis is better for the bipolar type. Answers A, C, and D are all true.

How well did you know this?
1
Not at all
2
3
4
5
Perfectly
59
Q

The Freedom from Distractibility Index is designed to:
Select one:

A.
assess working memory.

B.
assess crystallized intelligence.

C.
assess fluid intelligence.

D.
assess one’s ability to exert mental control.

A

The correct answer is D.

This index is designed to assess the ability to focus and exert mental control. Working memory (A) is an important aspect of performance and fluid intelligence, but this index is not designed to assess this. Crystallized intelligence (B) is associated with verbal and full scale IQ on intelligence tests and it is not usually impacted by contextual factors like environmental stimuli.

How well did you know this?
1
Not at all
2
3
4
5
Perfectly
60
Q

Individuals with autism spectrum disorder demonstrate impairments in language. All of the following language disturbances are associated with the disorder EXCEPT:
Select one:

A.
Reversal of pronouns.

B.
Echolalia.

C.
Overuse of metaphorical language.

D.
Abnormal rate and rhythm of speech.

A

The correct answer is C.

Individuals with autism spectrum disorder typically do not use metaphorical language. Answers A, B, and D are all language disturbances associated with autism spectrum disorder, which makes these answers incorrect.

How well did you know this?
1
Not at all
2
3
4
5
Perfectly
61
Q

Compensable factors identified within a job evaluation include all of the following EXCEPT:
Select one:

A.
Skill.

B.
Responsibility/accountability.

C.
Conscientiousness.

D.
Working conditions.

A

The correct answer is C.

Conscientiousness is not considered a compensable factor but instead is one of the traits in the Big Five factor theory. Skill (Answer A), responsibility/accountability (Answer B), and working conditions (Answer D) are all compensable factors utilized to identify value within a job evaluation.

How well did you know this?
1
Not at all
2
3
4
5
Perfectly
62
Q

What is the synchrony effect?
Select one:

A.
The concept that relationships act as a buffer against psychological and physical stress.

B.
The process of interacting with the world based on one’s existing perspective or schema.

C.
The concept that implies that the time of day individuals are tested for intelligence can greatly affect their performance.

D.
The process of changing one’s perspective or schema when new information is encountered.

A

The correct answer is C.

According to the synchrony effect, older adults tend to perform better when tested in the morning. Therefore, for testing to be meaningful, the time of day when individuals are examined should be taken into account. Answer A is incorrect, as it refers to the buffering hypothesis, which posits that relationships may act as a buffer in times of stress; however, the relationships should be perceived as valuable and adequate for the purpose. Answer B is incorrect, as it has to do with the Piagetian concept of assimilation. Answer D is incorrect, as it refers to the Piagetian concept of accommodation.

How well did you know this?
1
Not at all
2
3
4
5
Perfectly
63
Q

During a job selection process, a mistake occurred with the analysis of a certain predictive test for job success of applicants and, as a result, a number of employees were admitted into the company who the test predicted would not be successful. At the end of the first year, this particular group of employees was found to outperform the employees that the test predicted would be successful. This test produced which of the following?
Select one:

A.
True positives

B.
True negatives

C.
False positives

D.
False negatives

A

The correct answer is D.

The employees failed the predictor cutoff but exceeded the criterion cutoff, which is a false negative. If the employees exceeded both the predictor and criterion cutoff, it would be a true positive (answer A). If the employees failed both the predictor and criterion cutoff, it would be a true negative (answer B). If the employees exceeded the predictor cutoff but failed the criterion cutoff, it would be a false positive (answer C).

How well did you know this?
1
Not at all
2
3
4
5
Perfectly
64
Q

A new father is talking to his newborn son in baby talk. When the obstetrician comes in to speak to him, he talks in a normal voice. The father is displaying:
Select one:

A.
Stimulus generalization.

B.
Stimulus discrimination.

C.
Response discrimination.

D.
Response generalization.

A

The correct answer is B.

Stimulus discrimination includes the ability to differentiate among different stimuli or situations and act appropriately, depending on which is present. Stimulus generalization (answer A) is the opposite of stimulus discrimination and occurs when an individual performs the same behaviors regardless of whether the extant cues signal the availability or unavailability of reinforcement. Response discrimination (answer C) is not a proper psychological term, while response generalization (answer D) is a phenomenon in which reinforcement of some forms of behavior results in an increased probability not only of these forms but also of similar and non-reinforced forms.

How well did you know this?
1
Not at all
2
3
4
5
Perfectly
65
Q

According to Vygotsky’s theory, private speech is:
Select one:

A.
An egocentric, attention-seeking behavior.

B.
Equivalent to thought.

C.
A method young children use to regulate their behavior.

D.
A distraction to learning.

A

The correct answer is C.

Egocentric, attention-seeking behavior is not associated with Vygotsky’s model of cognitive development, which makes answer A incorrect. In Vygotsky’s model of cognitive development, Lev Vygostsky used the term “inner speech” to describe thought, but the term “inner speech” is not equivalent to thought, making answer B incorrect. Private speech is not a distraction but is essential for cognitive development, as it helps to integrate language with thought, which makes answer D incorrect.

How well did you know this?
1
Not at all
2
3
4
5
Perfectly
66
Q

Emotions, such as love and embarrassment, result from the context-specific __________ processing of signals from the __________.
Select one:

A.
cortical; subrosal cortex

B.
anterior cingulate; prefrontal cortex

C.
orbitofrontal; basal ganglia

D.
cortical; limbic system

A

The correct answer is D.

Emotional stimuli are registered by structures in the limbic system, communicating both directly and indirectly with the frontal cortex. Not only are direct signals sent up to the cortical areas for processing, but emotions, such as fear or passion, evoke the sympathetic nervous system. The bodily responses that follow, such as a quickened heartbeat, are interpreted by the frontal cortex, combined with direct input from the limbic system. Therefore, basic emotions, such as fear and pleasure, generated by the limbic system are interpreted as love and embarrassment due to the cortical processing of the signals from the limbic system in combination with context-specific information from the environment.

How well did you know this?
1
Not at all
2
3
4
5
Perfectly
67
Q

When is an evaluation of an employee’s physical fitness an acceptable part of an employer’s assessment program?
Select one:

A.
When the employee has recently recovered from an illness

B.
When such fitness is essential to successful performance of the job

C.
When an employee reaches the age of 55

D.
When both men and women are candidates for the same promotion opportunity

A

The correct answer is B.

In I/O psychology, the term assessment is most often used to describe methods used to evaluate and understand individuals. That outcome is then used to make decisions about individual performance, skill and training levels, aptitude, and organizational processes. The four means of assessment include norm-referenced tests, interviews, observations, and informal assessment procedures. In order to be useful, these measures must be valid and reliable. A fitness evaluation, which taps physical functioning, can be included in assessment procedures when the assessment is used for certain jobs requiring physical exertion (paramedics, military positions, police officers, etc.). An employer is not entitled to formally evaluate an employee’s fitness when the job does not require any special fitness, even if the employee is older or has recently been ill. Answer A is incorrect, as a fitness evaluation that is linked to recent illness could be viewed as an act that was preparatory to discrimination. The same is valid for Answer C, as age cannot justify an assessment. Answer D is incorrect; certainly, the fact that members of both genders are candidates for a promotion does not legitimize a fitness evaluation.

How well did you know this?
1
Not at all
2
3
4
5
Perfectly
68
Q

As they get older, infants detect affective states and share them with their parents. The child will use cues from the other person to deal with emotions. This is referred to as:
Select one:

A.
Social referencing.

B.
Mimicking.

C.
Object constancy.

D.
Imprinting.

A

The correct answer is A.

Social referencing refers to the customs and conventions that govern social interactions. An infant will use cues from another person, such as his or her mother, to deal with affective uncertainly and promote the development of self-regulation. Adaptation refers to the process by which organisms change to become more successful in a particular environment. Imprinting (answer D) refers to the bond between some newborn animals and their mothers.

How well did you know this?
1
Not at all
2
3
4
5
Perfectly
69
Q

Which of the following statements is false?
Select one:

A.
Terminal drop refers to a sudden decline in cognitive functioning before death.

B.
Some researchers suggest that the decline in cognitive ability is a result of the individual’s declining health status.

C.
Terminal drop is associated with Erikson’s last stage of developmental aging.

D.
Terminal drop may occur due to the onset of depression and withdrawal.

A

The correct answer is C.

Terminal drop is not traditionally associated with Erikson’s last stage of developmental aging. However, terminal drop does refer to a decline in intellectual functioning before death (answer A) and is thought to be associated with declining health (answer B). Alternative theories posit that the change in intellectual functioning is a result of depression (answer D).

How well did you know this?
1
Not at all
2
3
4
5
Perfectly
70
Q

Dr. Myers is seeing a client involved in a legal proceeding. The client’s attorney requests Dr. Myers’ expert testimony in the case. Which of the following is an appropriate response?
Select one:

A.
Temporarily pause treatment, testify as an expert witness, and then resume treatment at the close of the case

B.
Terminate therapy with the client and refuse to appear as an expert witness

C.
Continue therapy and also appear as an expert witness

D.
Continue therapy and decline to appear as an expert witness

A

The correct answer is D.

Psychologists cannot provide expert testimony to past (answer A) or current (answer C) therapy clients, as their objectivity will be impaired. Being asked to appear as an expert witness, however, does not mean that therapy should be terminated (answer B). Dr. Myers should kindly explain why providing expert testimony is not possible, and then resume the therapy.

How well did you know this?
1
Not at all
2
3
4
5
Perfectly
71
Q

On his way to work, Dr. Doom was killed in a car accident. Fortunately, he had a plan for his cases in the event he was severely disabled or killed. This means that he had probably:
Select one:

A.
Given another competent psychologist a key to his filing cabinet with patient records, along with written instructions regarding what to do in this event.

B.
Prepared a professional will where he diligently wrote all of his patients’ data and clinically relevant information, as well as what actions to take, in case something like this happened.

C.
Let his lawyer know what to do in case he was killed or disabled.

D.
Told his secretary that all of his clients should be given their records so that they can take them to their next treating therapist.

A

The correct answer is A.

Answer B is incorrect; it would be a good idea, with the exception of revealing patients’ data and clinical information, as this would not be a situation where confidentiality can be ethically broken. However, the idea of a professional will with a plan in case of a situation like this is great, when confidentiality is preserved. Answer C is incorrect. Again, this would be a good idea, but it is less realistic than answer A, as not all psychologists have personal lawyers. Answer D is not an ethical plan, as clients need more direction in their follow-up care; besides, to give them their records (especially without them asking for it) is typically not a good clinical decision.

How well did you know this?
1
Not at all
2
3
4
5
Perfectly
72
Q

The two phases of motivational interviewing are:
Select one:

A.
Resolving ambivalence and strengthening commitment to the decision for change.

B.
Creating motivational discrepancies and resolving ambivalence.

C.
Strengthening commitment to the decision for change and increasing motivation for change.

D.
Creating motivational discrepancies and increasing motivation for change.

A

The correct answer is C.

The two phases of motivational interviewing are strengthening commitment to the decision for change, and increasing motivation for change. Creating motivational discrepancies, resolving ambivalence, and eliciting self-motivational statements (answers A, B, and D) are all goals, rather than phases, of motivational interviewing.

How well did you know this?
1
Not at all
2
3
4
5
Perfectly
73
Q

Managers at UPscale, a packaging facility, installed sophisticated machines to assist the workers in package handling. The result was an increase in package production by more than 25%. The installation of such a machine in this facility is known as:
Select one:

A.
Mechanical assistance.

B.
Automated instillation.

C.
Automation.

D.
Automated package handling.

A

The correct answer is C.

Answers A, B, and D are distractors.

How well did you know this?
1
Not at all
2
3
4
5
Perfectly
74
Q

Dr. Hagen, an experimental psychologist, is conducting a study in which his laboratory animals will be euthanized as part of the procedure. Which of the following is true?
Select one:

A.
This procedure is unethical.

B.
This procedure is ethical.

C.
This procedure is considered ethical if the value of the research study outweighs the loss of animal life.

D.
The procedure is considered ethical if the person is trained to provide appropriate care for the animals involved in the experiment.

A

The correct answer is C.

Euthanizing animals in research is allowed if the scientific value of the research is sufficient to justify this practice. Answer A is therefore incorrect, and answer C is preferable to answer B. Further, the fact that a person is qualified to work with animals does not justify the use of lethality in research (answer D); further justification is needed.

How well did you know this?
1
Not at all
2
3
4
5
Perfectly
75
Q

Your client is agitated and discusses his plan to kill his wife and then himself. Your clinical judgment, based on your knowledge of your client, is that your best course of action in reducing the risk of harm is to maintain confidentiality and work with your client in the session to minimize the danger of your client acting on his plan. Your response to the situation is:
Select one:

A.
Illegal in all states, due to the Tarasoff ruling.

B.
Illegal in some states.

C.
Ethical only if you are correct and your client does not go through with the plan.

D.
Unethical.

A

The correct answer is B.

On the issue of legality, some but not all states require you to break confidentiality and warn authorities and the potential victim in this case. Remember that Tarasoff is case law, not state law; case law does not automatically translate into state law. Whether this action is ethical is subject to debate. However, the ethics of a response is based on the principles involved rather than the outcome.

How well did you know this?
1
Not at all
2
3
4
5
Perfectly
76
Q

The main goal of the Individuals with Disabilities Education Act is:
Select one:

A.
To ensure fair assessment of disabled children.

B.
To educate teachers about child development and disabilities.

C.
To provide disabled children with free and appropriate public education.

D.
To place disabled children in special education classes.

A

The correct answer is C.

As it describes the act’s main goal. Answer A is incorrect: assessment is a key component of the act, but it is not its main goal. Answer B is incorrect, as it is not part of the act. Answer D is incorrect, as it is incomplete; it describes an outcome of IDEA. Children with disabilities may or may not be determined to need placement in special education classes.

How well did you know this?
1
Not at all
2
3
4
5
Perfectly
77
Q

A four-year-old boy has been diagnosed with gender dysphoria. By adulthood, he will most likely:
Select one:

A.
Continue to meet diagnostic criteria for gender dysphoria and identify as heterosexual.

B.
Continue to meet diagnostic criteria for gender dysphoria and identify as homosexual or bisexual.

C.
No longer meet criteria for gender dysphoria and identify as heterosexual.

D.
No longer meet criteria for gender dysphoria and identify as homosexual or bisexual.

A

The correct answer is D.

At least three-fourths of young boys with diagnoses of gender dysphoria no longer meet criteria as adolescents or adults, while they do show homosexual or bisexual orientations. The other responses are inaccurate.

How well did you know this?
1
Not at all
2
3
4
5
Perfectly
78
Q

Several predictors are used to predict scores on a criterion. At the bivariate level, each predictor is highly correlated to the criterion and each predictor is highly correlated to each other. When all the predictors are used simultaneously to predict the criterion, only one of the predictors is still significant. What is responsible for this result?
Select one:

A.
Multicollinearity

B.
Criterion contamination

C.
Criterion deficiency

D.
Unreliable criterion

A

The correct answer is A.

Multicollinearity occurs when predictors are highly correlated, which results in predictors having little ability to predict the criterion. Criterion contamination (answer B) occurs when something unrelated to the criterion is affecting the criterion. Criterion deficiency (answer C) occurs when an important part of the construct is not being measured by the criterion. An unreliable criterion (answer D) would decrease the validity coefficient; however, in this case, the problem is the highly correlated predictors rather than the reliability of the criterion.

How well did you know this?
1
Not at all
2
3
4
5
Perfectly
79
Q

Rational Emotive Therapy was founded by:
Select one:

A.
Albert Ellis.

B.
Albert Bandura.

C.
Carl Rogers.

D.
Fritz Pearls.

A

The correct answer is A.

Albert Ellis (A) founded and developed Rational Emotive Therapy. Albert Bandura (B) is better known for social learning theories, Carl Rogers (C) with Rogerian or client-centered unconditional positive regard, and Fritz Perls (D) with Gestalt therapy.

How well did you know this?
1
Not at all
2
3
4
5
Perfectly
80
Q

The differences between shift work, compressed workweek, telecommuting, and flexible work schedules do NOT necessarily include:
Select one:

A.
A reduction in the actual hours worked.

B.
Changes in the level of productivity.

C.
Health issues.

D.
Transportation issues.

A

The correct answer is A.

This is the only factor that is not automatically affected by the type of work schedule and arrangement, among the ones listed above. Compressed workweek is a type of work schedule in which employees work a 40-hour workweek in fewer than five days with more than eight hours per day. Telecommuting is a work arrangement in which employees can work from home from anywhere they are. Flexible work schedules allow employees control over the time they report to work and the time they leave work on a given day (Landy & Conte, 2004). Answer B is incorrect, as productivity may be affected by the type of shift worked. Answer C is also incorrect, as health issues are a common consequence of shift work. Answer D is also incorrect, as telecommuting does not involve transportation; hence, this is a difference seen in at least one of the types of work schedules listed above.

How well did you know this?
1
Not at all
2
3
4
5
Perfectly
81
Q

Dr. Gleams practices Rational Emotive Therapy. A 26-year-old female patient comes to his private office and complains about unhappiness at work, in her primary relationship, and with her body weight. Dr. Gleams is likely to engage these problems by tapping into all of the following, except
Select one:

A.
replacing irrational thoughts with rational ones.

B.
gaining insight into the impact of early childhood experiences on current functioning.

C.
disputing long-held beliefs about the self and circumstances.

D.
identifying irrational beliefs that perpetuate emotional disturbance.

A

The correct answer is B.

Rational Emotive Therapy is focused on changing and disputing current irrational thinking patterns. While the self-indoctrination is rooted in childhood experiences, it does not focus on building insight into those experiences.

How well did you know this?
1
Not at all
2
3
4
5
Perfectly
82
Q

Avoidant personality disorder has a __________ correlation with extraversion and a __________ correlation with neuroticism.
Select one:

A.
positive; positive

B.
positive; negative

C.
negative; negative

D.
negative; positive

A

The correct answer is D.

Avoidant personality disorder is negatively linked with extraversion and positively linked with neuroticism. Answers A, B, and C are incorrect responses, portraying inaccurate correlations between avoidant personality disorder and extraversion and neuroticism.

How well did you know this?
1
Not at all
2
3
4
5
Perfectly
83
Q

Which group represents approximately 6% of the HIV-infected diagnoses?
Select one:

A.
African Americans

B.
Intravenous drug users

C.
Males

D.
Homosexuals/bisexuals

A

The correct answer is B.

Intravenous drug users represent Six percent (2,392) of HIV diagnoses in the United States were attributed to injection drug use (IDU) and another 3% (1,202) to male-to-male sexual contact and IDU (CDC, 2015).

How well did you know this?
1
Not at all
2
3
4
5
Perfectly
84
Q

Which of the following is the most accurate statement regarding the efficacy of Critical Incident Stress Debriefing?
Select one:

A.
Its efficacy has been proven unequivocally.

B.
It has proven to be ineffective.

C.
There are some studies showing efficacy and some that suggest that it is ineffective.

D.
There are no studies on the effectiveness of this technique.

A

The correct answer is C.

Debate continues over the effectiveness of Critical Incident Stress Debriefing, with some studies having found it effective and others ineffective. Efficacy has not been proven unequivocally (answer A) and the programs have not been found to be entirely ineffective (answer B), and there are definitely studies on efficacy (answer D).

How well did you know this?
1
Not at all
2
3
4
5
Perfectly
85
Q

Erikson’s second stage, autonomy vs. shame and doubt, emphasizes the importance of relationships with caregivers during the toddler years. According to Erikson, what would be the outcome of having failed to master this stage’s developmental task?
Select one:

A.
Perceiving the world as an unsafe place

B.
Inability to rely on others

C.
Pervasive dependency upon others

D.
Inadequate sense of agency and capability

A

The correct answer is C.

The developmental task of the autonomy vs. shame and doubt stage is to develop a good sense of self among others, including gaining a healthy sense of autonomy as well as of interdependence. Failure to master this stage would lead to a pervasive dependency upon others. Answer A is more related to the failure to master Erikson’s first stage, trust vs. mistrust. Answer B is incorrect, as it illustrates the opposite dynamic. Answer D is incorrect, as it refers to Erikson’s fourth stage, industry vs. inferiority. This stage emphasizes relationships with teachers and peers in school and other community figures. Successful resolution of the crisis between feeling competent and worrying about inadequacy leads to a sense of one’s self as a capable individual. Failure at resolution leads to paralyzing low self-esteem.

How well did you know this?
1
Not at all
2
3
4
5
Perfectly
86
Q

Post-concussion syndrome can be diagnosed without:
Select one:

A.
Insomnia or hypersomnia.

B.
Imaging evidence positive for brain insult.

C.
Impaired memory or attention.

D.
Irritability or aggression.

A

The correct answer is B.

PCS can be diagnosed even when brain imaging (e.g., MRI, CT) results do not show physical evidence of insult. PCS presents as a constellation of symptoms following a concussion. Cognitive symptoms associated with PCS often include memory and attention/concentration impairments, fatigue, insomnia, headaches, vertigo, dizziness, irritability, and aggressiveness. Anxiety, depression, subtle changes in personality, and apathy can also co-occur with PCS, as can visual and hearing impairments and anosmia (a loss of one’s sense of smell).

How well did you know this?
1
Not at all
2
3
4
5
Perfectly
87
Q

Overall, the number of psychological disorders that are diagnosed during adolescence are:
Select one:

A.
More than those found in adults.

B.
Less than those found in adults.

C.
About the same as those found in adults.

D.
Less than those found in children.

A

The correct answer is C.

Significant mental health problems are found in only about 20% of adolescents. In addition, these problems may have started before adolescence. Overall, the numbers of psychological disorders that are diagnosed in adolescence are similar to the numbers found in adulthood, making answers A, B, and D incorrect.

How well did you know this?
1
Not at all
2
3
4
5
Perfectly
88
Q

According to the buffering hypothesis, which of the following is most important?
Select one:

A.
The quantity of buffers

B.
The quality of buffers

C.
The frequency of exposure to buffers

D.
The intensity of the buffers

A

The correct answer is B.

For a buffer to be effective, it has been shown that the quality of the buffer is more important than the quantity of buffers. This means that people need to perceive the relationship as adequate for it to fulfill the purpose of buffering during times of stress. No matter how many relationships one has, if none are perceived as qualitatively adequate, relationships will not serve as a buffer during stress. Hence, answers A, C, and D are incorrect, as the quantity of relationships, the frequency of contact, and their intensity are not important for a buffer to be effective.

How well did you know this?
1
Not at all
2
3
4
5
Perfectly
89
Q

The purpose of the second stage in administering the Stanford-Binet is to:
Select one:

A.
Select the starting point on the test.

B.
Assess fluid intelligence.

C.
Complete all appropriate subtests.

D.
Establish a basal level and a ceiling level for each test.

A

The correct answer is D.

Response A is associated with the first stage of administration. B is an important aspect of intelligence, but the second stage is designed to identify basal and ceiling levels. C is incorrect, as all subtests are used.

How well did you know this?
1
Not at all
2
3
4
5
Perfectly
90
Q

When a participant tries to understand the purpose of the study and adjusts his or her behavior accordingly, which of the following has occurred?
Select one:

A.
Regression

B.
Selection

C.
Maturation

D.
Demand characteristics

A

The correct answer is D.

Demand characteristics refer to participants changing their behavior when they believe they have deciphered the purpose of the study. Regression (answer A) refers to participants who have extreme scores tending to score closer to the mean on subsequent assessment. Maturation (answer C) refers to changes in participants that have occurred naturally. Selection (answer B) refers to participants in groups being dissimilar.

How well did you know this?
1
Not at all
2
3
4
5
Perfectly
91
Q

Which of the following characteristics are associated with reading disorder?
Select one:

A.
The individual has difficulty with reading accuracy, speed, or comprehension.

B.
The individual has difficulty with speed and fluency in reading.

C.
The individual has difficulty with fluency and pronunciation in reading.

D.
The individual has difficulty with accuracy and perceptual problems in reading.

A

The correct answer is A.

Reading disorder is a learning disorder in which a person has difficulty with reading accuracy, speed, or comprehension (answer A).

92
Q

__________ has a limited capacity but is capable of holding and manipulating information for a short period of time.
Select one:

A.
Long-term memory

B.
Primary memory

C.
Semantic memory

D.
Implicit memory

A

The correct answer is B.

Primary memory is an archaic term that was used to describe the first level of memory in the dual-store model of memory. The term and its underlying concepts were later replaced with short-term memory. It is a place for incoming information to be held and manipulated before being encoded into secondary memory (the second level of memory in the dual-store model). Long-term memory (answer A) refers to memory that lasts for a relatively long period of time, at least several minutes. Semantic memory (answer C) is a type of explicit memory (or declarative memory) involved in remembering factual information. Implicit memory (answer D) refers to the retrieval of information from past experience, in the absence of conscious awareness that such memory retrieval has occurred.

93
Q

The most common adult disorder that begins in adolescence is NOT:
Select one:

A.
Anxiety disorders.

B.
Mood disorders.

C.
Drug and alcohol abuse/dependence.

D.
Oppositional defiant disorder.

A

The correct answer is D.

Oppositional defiant disorder has a typical onset age of eight. In contrast, anxiety disorders (answer A), mood disorders (answer B), and drug and alcohol disorders (answer C) often emerge during adolescence. Despite commonly having an age of onset in adulthood, these disorders are common among adolescents.

94
Q

Which of the following disorders is least likely to be associated with the development of schizoaffective disorder?
Select one:

A.
histrionic personality disorder

B.
schizotypal personality disorder

C.
borderline personality disorder

D.
paranoid personality disorder

A

The correct answer is A.

Histrionic personality disorder is less likely to be associated with schizoaffective disorder than schizotypical, borderline, and paranoid personality disorders. This makes answers B, C, and D incorrect.

95
Q

Avoidant personality disorder is least likely to be comorbid with:
Select one:

A.
borderline personality disorder.

B.
dependent personality disorder.

C.
paranoid personality disorder.

D.
antisocial personality disorder.

A

The correct answer is D.

Avoidant personality disorder is not at all likely to be comorbid with the extremely externalizing condition known as antisocial personality disorder. It shares some limited features, however, with borderline, dependent, and paranoid personality disorders.

96
Q

The correct answer is D.

Avoidant personality disorder is not at all likely to be comorbid with the extremely externalizing condition known as antisocial personality disorder. It shares some limited features, however, with borderline, dependent, and paranoid personality disorders.The Buckley Amendment (1974) is also known as:
Select one:

A.
Wickline vs. California.

B.
Family Educational Right of Parameters Act.

C.
Confidentiality of Medical Information.

D.
Family Educational Right to Privacy Act.

A

The correct answer is D.

The Buckley Amendment and FERPA refer to the same law. Ensure that you are familiar with both terms. Answers A, B, and C are not associated with the Buckley Amendment and are therefore incorrect.

97
Q

A client’s adaptive behavior is initially rated by:
Select one:

A.
The client.

B.
The psychologist.

C.
The client’s caretaker.

D.
Both the psychologist and the client’s caretaker.

A

The correct answer is B.

This rating is part of the psychotherapy and assessment process provided by the psychologist. A and C may be involved through self-report measures, but the main assessment is done by the psychologist using scales like the VABS or the AAMR Adaptive Behavior Scales.

98
Q

Norepinephrine is identified as the primary cause of depression in the __________ hypothesis.
Select one:

A.
neurotransmitter

B.
adrenergic

C.
catecholamine

D.
autonomic

A

The correct answer is C.

Norepinephrine is identified as the primary cause of depression in the catecholamine hypothesis. The catecholamine hypothesis is supported by pharmacological studies that have shown that both MAOIs and TCAs are efficacious in treating depression via their effects on norepinephrine.

99
Q

Which of the following tests is used to classify TBI?
Select one:

A.
Color-word Stroop test

B.
GCS

C.
WAIS-IV and WISC-V

D.
WCST

A

The correct answer is B.

The GCS is the most commonly used system for classifying TBI severity. The color-word Stroop test (answer A) is used to measure behavioral inhibition and selective attention. The WAIS-IV and WISC-V (answer C) measure crystallized and fluid intelligence. The WCST (answer D) measures shifting mental set and problem-solving.

100
Q

Which of the following is least true?
Select one:

A.
The APA Ethics Code is to be considered “higher” than the law.

B.
As psychologists, we aspire to a higher standard than that proposed by the law.

C.
Psychologists don’t have to clarify their commitment to the Ethics Code when it is in conflict with the law.

D.
Psychologists can make an individual decision not to follow the law when it is in conflict with the Ethics Code.

A

The correct answer is C.

When considering the Ethics Code and the law, a psychologist must consider the Code “higher” than the law (A), as psychologists seek to aspire to a higher standard of conduct than that proposed by the law (B). A psychologist is allowed to make an individual decision when there is a conflict between the Code and the law (D), and can choose to uphold an Ethical Standard. However, a judge can hold the psychologist legally responsible for this, and charge the psychologist with a legal infraction. Answer C is the “least correct” answer to this question, as a psychologist is required to clarify their commitment to the Ethics Code when there is a conflict between the Code and the law.

101
Q

Key issues that feminist therapy theory focuses on include:
Select one:

A.
Active counseling and giving information.

B.
Pluralism and personal validation.

C.
Using community resources.

D.
Social context and oppression.

A

The correct answer is D.

Feminist therapy may be active, with the therapist serving as an advocate for the client and giving information (answer A). It may promote pluralism, teaching the client multiple ways to view the world, and personal validation (answer B). Feminist therapy may use community resources (answer C). However, the key issue addressed throughout the therapy and guiding the application of the above concepts is that the individual’s problems arise in a social context due to oppression (answer D).

102
Q

Which of the following is NOT a characteristic of RAD?
Select one:

A.
Limited positive affect

B.
Lack of reticence in approaching unfamiliar adults

C.
Minimal socio-emotional responsiveness to others

D.
Unexplained sadness, irritability, or fearfulness, even in nonthreatening or pleasant situations

A

The correct answer is B.

A pattern of interacting freely with unfamiliar adults is indicative of disinhibited social engagement disorder. Answers A, C, and D are all characteristics of RAD.

103
Q

The Little Albert experiment is an example of:
Select one:

A.
Operant conditioning.

B.
Vicarious conditioning.

C.
Habituation.

D.
Pavlovian conditioning

A

The correct answer is D.

Little Albert was repeatedly presented with a white lab rat, followed by a loud noise; the loud noise was a natural fear-evoking stimulus at the beginning of the experiment but the white lab rat was not. After several pairings, the white lab rat became a fear-evoking CS. Therefore, the learning involved in the experiment was an example of Pavlovian (or classical) conditioning (answer D). Operant conditioning (answer A) involves making reinforcement contingent on the performance of a particular voluntary behavior, and the fear response that was conditioned in Little Albert was not a voluntary behavior. Vicarious conditioning (answer B) is a distractor. To have decreased responsiveness to a stimulus is habituation, not operant (or Pavlovian) conditioning, making answer C incorrect.

104
Q

Group cohesiveness is associated with:
Select one:

A.
Independent thinking.

B.
Better performance in individual members.

C.
Better performance with people-oriented leaders.

D.
Worse performance.

A

The correct answer is C.

Cohesion is the degree to which team members desire to remain in the team and are committed to team goals. Cohesiveness is associated with improved performance when leaders are people-oriented and leadership is supportive (Tziner & Vardis, 1982). Cohesiveness often facilitates group decision-making because members feel more open and secure in their communications with one another. However, because cohesiveness also includes strong norms and the pressure for conformity (Landy & Conte, 2004), there is a risk of developing a groupthink mentality.

105
Q

Symptoms of hypersexuality, visual agnosia, and emotional placidity are associated with which of the following disorders?
Select one:

A.
Korsakoff’s syndrome

B.
Gerstmann’s syndrome

C.
Klüver-Bucy syndrome

D.
Landau-Kleffner syndrome

A

The correct answer is C.

Klüver-Bucy syndrome is associated with bilateral damage to the amygdalae. It is associated with hypersexuality, visual agnosia, lack of emotional response, docility, apathy, dietary changes, and mouthing of objects. Typically not all of these symptoms are present in an individual with Klüver-Bucy syndrome. Answer A is incorrect, as Korsakoff’s syndrome is associated with anterograde and retrograde amnesia and is caused by vitamin B1 deficiency (typically associated with alcoholism). Answer B is incorrect, as Gerstmann’s syndrome is associated with dysgraphia, dyscalculia, an inability to distinguish right and left, and an inability to identify fingers. It is caused by damage to the left parietal lobe, typically due to stroke. Answer D is also incorrect, as Landau-Kleffner syndrome is a rare childhood neurological disorder characterized by typical development up to about 5-7 years of age, followed by the gradual or sudden inability to understand or express language.

106
Q

Which of the following is NOT a symptom of alcohol withdrawal?
Select one:

A.
Increased heart rate

B.
Perceptual disturbances

C.
Psychomotor retardation

D.
Nausea

A

The correct answer is C.

Agitation (rather than psychomotor retardation) is typically associated with alcohol withdrawal. It helps to remember that a withdrawal syndrome usually leads to symptoms that are the opposite of those induced by the substance itself. Interestingly, however, nausea (response D) can accompany both acute alcohol intoxication and alcohol withdrawal.

107
Q

Camilla is in dire need of psychological help after discovering, during the same week, that she is pregnant and that her husband Charles has been involved in an extramarital affair for several years. Camilla does not currently work, and since she depended on Charles for over two decades, her financial resources are almost nonexistent. In addition, her health insurance does not cover psychological services. She is experiencing high distress, hopelessness, and confusion about being pregnant. She is therefore in a very difficult situation, in addition to experiencing intense emotional pain due to being cheated on and betrayed by her husband. As Camilla is versed in the arts and interior design, she finds a counselor and proposes a bartering arrangement whereby she offers her services in exchange for therapy sessions. Which of the following choices is an incorrect course of action for her psychologist to take?
Select one:

A.
The psychologist declines the proposal and refers the client to a sliding scale/no-fee community clinic.

B.
Given the client’s dire situation and the APA’s encouragement regarding engagement in pro bono work, the psychologist accepts the bartering arrangement.

C.
The psychologist agrees to see the client but declines the bartering arrangement and offers to see Camilla free of charge.

D.
The psychologist refuses the arrangement without giving Camilla any explanation.

A

The correct answer is B.

Answer B is an incorrect interpretation of pro bono work. Bartering and pro bono are different arrangements, as the former entails some form of payment, while the latter does not; therefore, the sentence is incorrect. Answers A, C, and D are all correct choices and possible scenarios. Answer C would be the preferred course of action and is a correct description of pro bono work, which is ethical and encouraged when it is administered appropriately and does not involve insurance. Answer A is a very appropriate follow-up for this client, and answer D is also a possibility, though it would not be encouraged by the Ethics Code, given the highly distressed state of the client.

108
Q

Dissociative identity disorder involves an inability to recall important personal information that goes beyond what would be expected of normal forgetfulness. Which type of identity state demonstrates the least severe amnesia?
Select one:

A.
The primary identity

B.
Hostile identities

C.
Passive identities

D.
Child-like identities

A

The correct answer is B.

The more assertive and hostile identities are typically more aware of (i.e., have less amnesia for) other identities than the primary identity (answer A). The primary identity, or host, tends to be more passive and dependent, and hence more amnesic, as it may be a child-like identity; this makes answers C and D incorrect.

109
Q

Making a legal decision based on the “best interests of the child” is most related to:
Select one:

A.
Child custody.

B.
Academic placement.

C.
Whether to release information when abuse is suspected.

D.
A way to determine treatment modality.

A

The correct answer is A.

In legal proceedings, “best interests of the child” most often relates to custody proceedings. Though you would want the best interest of the child in general, this specific terminology is used regarding child custody issues. This specific terminology is not used in academic placement, the release of information regarding abuse (due to the fact that therapists are mandated reporters), or choices in treatment using a modality, which makes answers B, C, and D false.

110
Q

Common symptoms of acute withdrawal from benzodiazepines include:
Select one:

A.
Increased anxiety, perspiration, and tachycardia.

B.
Slurred speech and sedation.

C.
Mania.

D.
Trichotillomania.

A

The correct answer is A.

These are symptoms of acute withdrawal from benzodiazepines. Answer B is incorrect, as these are the effects of excessive sedation. Answer C is incorrect, as mania is not a symptom of acute benzodiazepine withdrawal, although agitation may occur. Answer D is also incorrect, as it refers to a hair-pulling disorder.

111
Q

The sample variance is always:
Select one:

A.
Positive.

B.
Negative.

C.
Large.

D.
Small.

A

The correct answer is A.

The sample variance is always a positive value, as it is calculated as the sum of the squared deviations divided by N - 1. The variance may be larger or smaller depending on the spread of the distribution.

112
Q

Catell’s 16-factor system was used in the development of:
Select one:

A.
The Big Five personality inventory.

B.
The Rorschach inkblot test.

C.
The Myers-Briggs test.

D.
Beck’s anxiety scale.

A

The correct answer is A.

Catell’s 16-factor system and Eysenck’s model, based on language research, served as the basis for the Big Five personality model, which is another lexical approach. Answers B, C, and D are not related to the 16-factor system, which makes these answers incorrect.

113
Q

Approximately __________ cases of child sexual abuse are reported each year.
Select one:

A.
100000

B.
200000

C.
500000

D.
1000000

A

The correct answer is B.

Child sexual abuse is a common problem in society. The number of reported cases of child sexual abuse per year is 200,000 (answer B). It is possible to overestimate the incidence of child sexual abuse, but 1,000,000 (answer D) and 500,000 (answer C) cases would be high rates of reporting and unusual for these types of situations. While 100,000 (answer A) is close, it is too low for the actual annual incidence rate.

114
Q

In conduct disorder, females are more likely than males to:
Select one:

A.
Have comorbid ADHD.

B.
Be classified as having childhood-onset type.

C.
Engage in physical altercations.

D.
Run away from home.

A

The correct answer is D.

Females with conduct disorder are more likely than males to engage in non-violent forms of behavior, such as running away from home (it is noteworthy that such behavior in females may well be in response to sexual abuse at home). Answer B is incorrect, as males are in a 5:1 ratio compared to females for the childhood-onset type. Answer A is incorrect, as both boys and girls with conduct disorder are likely to have comorbid ADHD. Answer C is incorrect, as males are more physically violent than females across the lifespan.

115
Q

Which part of the brain does NOT work properly in a person with Prader-Willi syndrome?
Select one:

A.
Basal ganglia

B.
Temporal lobe

C.
Hypothalamus

D.
Hippocampus

A

The correct answer is C.

With Prader-Willi syndrome, several proteins that affect the functioning of the hypothalamus are not produced. The hypothalamus is responsible for the regulation of food and hunger. The basal ganglia is responsible for motor control and learning, working in conjunction with the cerebral cortex and the thalamus to shift activities and make choices, which makes answer A false. The temporal lobe processes auditory information and is also involved in memory, personality, and behavior, which makes answer B false. The hippocampus (answer D) controls memory and spatial navigation, and is also part of the limbic system, which is responsible for emotional responses, motivation, and memory.

116
Q

A manager suspects that one of his supervisors is exhibiting the severity bias because of the supervisor’s evaluations of his/her supervisees. What would make the manager suspect this?
Select one:

A.
The supervisor’s evaluations are systematically and consistently low across supervisees.

B.
The supervisor’s evaluations are systematically and consistently high across supervisees.

C.
The supervisor’s evaluations indicate that none of his/her supervisees is average on any domain of assessment.

D.
The supervisor has received poor evaluations himself/herself.

A

The correct answer is A.

Severity error is the tendency of some raters to give unusually low ratings (A). The Leniency bias is the tendency of some raters to give unusually high ratings (B). Failure to make average ratings (C) does not rule out the possibility that very good ratings were made in addition to bad ratings, the former being inconsistent with a severity bias. There is no evidence that a poor evaluation leads a supervisor (D) to be biased toward extreme negative judgments of others.

117
Q

Criterion deficiency occurs when:
Select one:

A.
conceptual criterion has been omitted from the actual criterion measure.

B.
extraneous information has been added to the actual criterion measures.

C.
scores from the criterion measure are not consistent over time.

D.
scores from the criterion measure are unrelated to performance.

A

The correct answer is A.

Criterion Deficiency occurs when the assessment of the conceptual criterion, e.g., job performance, is made using criteria measures that are irrelevant. The actual criteria used may not overlap with the conceptual criterion, and important relevant criteria are not used leading to a distorted understanding of the conceptual criterion. The actual criteria set is thus an incomplete representation of what one is trying to assess.

118
Q

A decision-making group that fails to consider alternatives, experiences pressure to reach consensus, and is threatened by external sources is likely to be engaged in:
Select one:

A.
Groupthink.

B.
Group polarization.

C.
Group facilitation.

D.
Social loafing.

A

The correct answer is A.

Groupthink is the tendency for individuals within a group to disregard different options in the interest of achieving agreement. Consensus is considered to be more important than rational consideration. Groups engaging in groupthink show the following cognitive impairments: They do not consider all alternatives, they do not conduct careful and unbiased searches for relevant information, and they selectively process available information. Groups that fall into the groupthink trap exhibit telltale signs, such as pressure toward unanimity, collective rationalization, strong pressures to conform, and self-appointed censorship of dissenting views. They sometimes develop a siege mentality wherein external sources are thought of as threatening, and they can develop a false sense of morality. Group polarization (Answer B) refers to the tendency of group attitudes to polarize after group discussion. Social facilitation (Answer C) and social loafing (Answer D) refer to the effects of proximal others on one’s performance. In the case of social facilitation, the presence of others can facilitate performance of well-practiced tasks and inhibit the performance of novel tasks. Social loafing (Answer D) refers to the reduction of effort by team members engaged in the same task, as responsibility for performance is spread among members.

119
Q

The genetic disorder that results in the body’s inability to process an essential amino acid and often results in mental retardation is called:
Select one:

A.
Phenylkentonuria.

B.
Down Syndrome.

C.
Cystic fibrosis.

D.
Tay-Sachs disease.

A

The correct answer is A.

Phenylketonuria (PKU) is a genetic disorder that causes people to be unable to physically process phenylalanine, an amino acid necessary for the body to run well. Down syndrome is caused by the presence of an extra 21st chromosome. Cystic fibrosis is a recessive disorder that results in excess mucus throughout the body. Tay-Sachs disease is also a recessive trait disorder that results in brain degeneration.

120
Q

__________ is the result of including extraneous variables (i.e., variables unrelated to the conceptual criterion being assessed) in a criterion measure.
Select one:

A.
Criterion deficiency

B.
Criterion contamination

C.
Criterion relevance

D.
Actual criterion

A

The correct answer is B.

A conceptual criterion is the theoretical construct one wishes to understand, and the actual criterion is the actual variable measured to assess that construct. For example “academic success” is a conceptual criterion that might be assessed using the actual criterion“Grade Point Average.” Often the conceptual criterion is assessed with a number of actual criteria measures. Criterion Contamination occurs when the assessment of the conceptual criterion, e.g., job performance, is made not only by relevant actual criteria measures, but also in terms of actual criteria that could be considered extraneous. Criterion deficiency occurs when the assessment of the conceptual criterion includes no actual criterion measures that are relevant to the conceptual criterion. Criterion relevance occurs when the assessment of the conceptual criterion includes actual criterion measures that are fully relevant to the conceptual criterion.

121
Q

Which of these was NOT mentioned as a ruled-out alternative explanation for the predictive accuracy advantage of actuarial data over clinical judgment?
Select one:

A.
Examiner’s field of training

B.
Examiner’s demographic data

C.
Length of experience

D.
Task-related experience

A

The correct answer is B.

In a landmark study comparing actuarial data to clinical judgment, Grove and Meehl demonstrated that, in general, actuarial data are equal to or better than clinical judgments. To examine possible alternative explanations, they reviewed and ruled out common contributors to bias in this literature, including examiner’s field of training (answer A), length of experience (answer C), and task-related experience (answer D). They did not rule out examiner’s demographic data (answer B).

122
Q

Which of the following factors is most likely to be associated with a good prognosis in schizophreniform disorder?
Select one:

A.
Flattened affect

B.
Confusion during the height of the psychotic episode

C.
Poor school and work history

D.
Living in a developed nation

A

The correct answer is B.

Confusion during the height of a psychotic episode is a good prognostic sign, whereas negative/deficit symptoms (e.g., flattened affect; answer A), poor premorbid functioning (answer C), and living in a developed nation (answer D) are all associated with poor outcome.

123
Q

According to research, by what age does social referencing begin?
Select one:

A.
16 months

B.
12 months

C.
22 months

D.
Birth

A

The correct answer is B.

Social referencing becomes common before one year of age, when the infant uses cues from another person, most likely the mother, to deal with affective uncertainty.

124
Q

In Moreno’s psychodrama, the members of the group who assume the roles of significant others in the drama are called:
Select one:

A.
Protagonists.

B.
Agonists.

C.
Auxiliary egos.

D.
The audience.

A

The correct answer is C.

The auxiliary egos are the actors in the drama who represent significant others. Protagonists (answer A) are key actors who represent the theme of the drama. The audience (answer D) consists of the individuals who witness the drama and represent the world at large. Agonists (answer B) are substances that cause a biological action.

125
Q

Which of these is NOT part of EMDR treatment?
Select one:

A.
Logging triggers for distressing recollections

B.
Cognitive reprocessing

C.
Desensitization

D.
Flooding

A

The correct answer is A.

EMDR is a treatment focused on the desensitization (answer C) and cognitive reprocessing (answer B) of traumatic memories. It accomplishes this through several phases including intake, preparation, saccadic eye movements during recollection of traumatic images, and instillation of positive cognitions. By focusing on the traumatic memories, it also serves as a form of flooding (answer D). In contrast to answer A, the client is asked to log distressing memories and relaxation practice rather than the triggers for those memories.

126
Q
Danny, a seven-year-old boy, clings to his mother whenever she drops him off at school. When he arrives in the classroom, he tends to stay close to his teacher. Danny rarely raises his hand in class and never volunteers to read out loud. He generally refuses to participate in group play, often blushing or even crying if the issue is forced. Danny would most likely be diagnosed with:
Select one:

A.
separation anxiety disorder.

B.
reactive attachment disorder.

C.
social anxiety disorder.

D.
generalized anxiety disorder.

A

The correct answer is C.

These are the common features of social anxiety disorder (formerly social phobia) in children. Answer A is incorrect because the primary fear does not relate to safety of his parent figures or himself (note that clinging to parents and school refusal can manifest in social anxiety disorder, not just separation anxiety disorder). Answers B and D are incorrect because Danny’s fear centers on social interactions at school.

127
Q

Matthew, a 20-year-old man, is brought in for assessment. He tells you he has been convicted of sexual assault. When you ask Matthew how he thinks his victim feels, he shrugs and comments, “If she didn’t want to have sex, she shouldn’t have been dressed the way she was.” Matthew has been unable to hold down a job for the past three years. When asked how he meets his financial obligations, Matthew replies with a smirk, saying, “I can usually guilt my mom into bailing me out.” As the assessment progresses, you notice that there are multiple inconsistencies between what Matthew is saying and the information you have received from collateral sources. You would most likely diagnose Matthew with:
Select one:

A.
borderline personality disorder.

B.
narcissistic personality disorder.

C.
antisocial personality disorder.

D.
oppositional defiant disorder.

A

The correct answer is C.

Matthew displays many of the core features of antisocial personality disorder. Answer A is incorrect, as there is no indication of the emotional lability characteristic of borderline personality disorder. Answer B is incorrect because narcissism is supplanted by the severe history of antisocial actions. Answer D is incorrect, as the behavior patterns far transcend oppositionality, and ODD is largely a diagnosis of childhood and adolescence.

128
Q

According to the APA Code of Ethics (2002), which of the following is a mandated responsibility of supervisors?
Select one:

A.
Provide clients

B.
Provide payment

C.
Provide feedback

D.
Meet licensing requirements mandated by the state

A

The correct answer is C.

Providing feedback is a mandated supervisory responsibility according to the APA Ethics Code. Some supervisors may assist supervisees in obtaining clients (answer A) but this is not a mandated responsibility. Providing payment and meeting licensing requirements mandated by the state are not mandatory supervisory responsibilities according to the APA Ethics Code, which makes answers B and D incorrect.

129
Q

The autokinetic effect is:
Select one:

A.
The likelihood that people will dance with more gusto if others are dancing nearby.

B.
A socially facilitated and false pretension to enjoy cars so as to better bond with other males.

C.
The likelihood that a group of people in a darkened room with an affixed small dot of light will begin to agree that it is moving.

D.
The bond between family members, as contrasted with non-blood-related family relationships.

A

The correct answer is C.

The autokinetic effect is actually the likelihood that a group of people in a darkened room with an affixed small dot of light will begin to agree that it is moving. All of the other answers have nothing to do with Sherif’s study of the autokinetic effect, or the autokinetic effect in general.

130
Q

Of the different decision-making styles, which is viewed as the most effective?
Select one:

A.
Intuitive

B.
Planning

C.
Impulsive

D.
Delaying

A

The correct answer is B.

Tiedeman and O’Hara are known for their career development model that emphasized cognitive development, responsibility for making choices, and vocation as an element of one’s ego identity. Within this, they emphasized two processes, namely, differentiation and integration. Differentiation refers to making distinctions between different aspects of oneself and one’s environment. Integration refers to unifying these different aspects, resulting in making better decisions, more refined goals, and developing useful plans. The seven stages of decision making are exploration, crystallization, choice, clarification, induction, reformation, and integration. The styles of decision-making are planning (Answer B), intuitive (Answer A), impulsive (Answer C), agonizing, delaying (Answer D), paralytic, fatalistic, and compliant. Of these decision-making styles, planning is the most effective.

131
Q

Thorndike’s instrumental learning is based on the idea that:
Select one:

A.
Unconditioned stimuli increase the occurrence of the behaviors that produce them.

B.
Unpleasant consequences decrease the occurrence of desirable behavior, while pleasant consequences decrease the occurrence of undesirable behavior.

C.
Negative outcomes increase the occurrence of undesirable behavior.

D.
Unpleasant consequences decrease the occurrence of behavior, while pleasant consequences increase the occurrence of behavior.

A

The correct answer is D.

Answer D is basically what is stated in the law of effect, which provides the experimental foundation for instrumental learning. Answers B and C are incorrect because instrumental learning does not make different predictions about the effects of consequences on desirable or undesirable behavior. Answer A is incorrect because unconditioned stimuli are part of the theory of classical (or Pavlovian) conditioning rather than instrumental learning.

132
Q

Dr. Joyce wishes to raise her probability for correctly rejecting a null hypothesis concerning the mean difference between a treatment group and control group. She should select:
Select one:

A.
A one-tailed test instead of a two-tailed test.

B.
A two-tailed test instead of a one-tailed test.

C.
A one-way ANOVA instead of a two-way ANOVA.

D.
A two-way ANOVA instead of a one-way ANOVA

A

The correct answer is A.

A one-tailed test has a greater chance of rejecting the null hypothesis because the region for rejection is larger than in a two-tailed test. Because there is only one independent variable (group membership), a two-way ANOVA would not be appropriate.

133
Q

All of the following are listed in Meyer’s minority stress model as types of proximal stressors experienced by gay or lesbian individuals EXCEPT:
Select one:

A.
Identity confusion.

B.
Expectation of rejection.

C.
Concealment of sexual orientation.

D.
Internalized homophobia.

A

The correct answer is A.

Identity confusion is the first state in Cass’s model (1979) of gay and lesbian identity formation and is not one of the three proximal stressors listed in Meyer’s minority stress model. In contrast, expectation of being rejected (answer B), concealment of sexual orientation (answer C), and internalized homophobia (answer D) are the three proximal (internal) stressors listed by Meyer.

134
Q

You complete a psychological assessment and the client stated that she has found this helpful in managing her psychological difficulties. The client asks to see you in therapy because she established a good rapport with you during the testing. By taking on this client for therapy, you are acting:
Select one:

A.
Ethically only if your objectivity is not impaired.

B.
Unethically.

C.
Ethically only if the psychological assessment occurred at least two years prior to the referral.

D.
Ethically only if the client shows serious symptoms during psychological testing and there is concern for well-being.

A

The correct answer is A.

Establishing good rapport with a client during psychological testing and then taking on the same client in psychotherapy is an example of a multiple relationship that is not harmful. The first role as evaluator actually enhances the second role as therapist because the client had a good rapport with the psychologist. Answer B is incorrect. Answers C and D are distractors.

135
Q

What type of assessment, used with children with learning disabilities, is based on Vygotsky’s theory?
Select one:

A.
Curriculum-based assessment

B.
Performance-based assessment

C.
Dynamic assessment

D.
Learning-potential assessment

A

The correct answer is C.

Dynamic assessment attempts to measure not only the products or processes of learning but also the potential for learning. It focuses on the difference between latent capacity and developed ability (that is, the extent to which developed abilities reflect latent capacity). It seeks to measure the psychological processes involved in learning and change rather than the products of these processes by presenting sequences of progressively more challenging tasks and providing continuous feedback on performance in an atmosphere of teaching and helping (that is, scaffolding) that guides the test-taker toward the right answer. Answer A is incorrect, as it refers to the observation and recording of a student’s performance based on the local curriculum, for the purpose of gathering information to make instructional decisions. Answer B is incorrect. This type of assessment provides teachers with information about how a student understands and applies knowledge. Answer D is incorrect: Learning potential assessment is used for dynamic testing of children, adolescents, and adults for potential cognitive growth.

136
Q

An 18-year-old male arrives for therapy smelling of alcohol and tells his therapist that he is going to “end it all” and hang himself while his parents are in Africa this week. He refuses to go to the hospital. Whom should the therapist call first?
Select one:

A.
His Alcoholics Anonymous sponsor

B.
Child Protective Services

C.
The hospital for an involuntary admission

D.
A designated mental health professional to assess the client

A

The correct answer is D.

The PET or police must be called first, as involuntary hospitalization is warranted given the threat. Calling his Alcoholics Anonymous sponsor (answer A) does not deal with the immediate emergency. Child Protective Services (answer B) should not be called, as no minors are involved. The hospital does not admit patients involuntarily without an assessment (answer C) by a designated mental health professional. The police may also be called instead of a designated mental health professional, but this step is usually taken only if the client must be located.

137
Q

__________ refers to extensive periods of sadness accompanied by disruptions in eating, sleeping, and social habits.
Select one:

A.
Schizophrenia

B.
OCD

C.
Major depression

D.
Paraphilia

A

The correct answer is C.

Cognitive, behavioral, and physical symptoms, including disruptions in eating, sleeping, and social behaviors and sadness or numbness, are present in major depression. Schizophrenia (answer A) is primarily characterized by psychotic symptoms regardless of mood state. Obsessive-compulsive disorder (OCD; answer B) is characterized by intense and intrusive thoughts or images that increase anxiety, which is alleviated when the person engages in repetitive behaviors. Paraphilias (answer D) are disturbances in sexual impulses and include pedophilia and exhibitionism.

138
Q

Anne watched a movie with a group of friends. Anne did not like the movie. After the movie, the group talked about how they felt about it. All of members agreed that it was bad. In fact, they all agreed that it was a total waste of time and money. What group phenomenon was at work here?
Select one:

A.
Conformity

B.
Deindividuation

C.
Group contagion

D.
Group polarization

A

The correct answer is D.

Group polarization was at work in this story because Anne initially felt like the movie was not a good movie, but when she got in a group her inclination was not only agreed upon, it was strengthened to the point where all the group members felt like the movie was a total waste of money. Conformity is a change in behavior or belief as a result of real or imagined group pressure, which makes answer A incorrect. Deindividuation is a loss of self-awareness and evaluation apprehension, which makes answer B incorrect. Answer C is incorrect because group contagion is transmission of emotion through a crowd or group but not the strengthening of average inclination.

139
Q

A 6-year-old child begins to demonstrate severe deficits in cognitive, emotional, and social abilities. Her parents commented that the child has always seemed distant, not making eye contact when she was a baby and seemingly disinterested with playing with others her age. Since starting school, the difference between her and other children is more apparent. This child would be a candidate for which diagnosis?
Select one:

A.
Childhood disintegrative disorder

B.
Asperger’s disorder

C.
Pervasive developmental disorder

D.
Autism spectrum disorder

A

The correct answer is D.

Answers A, B, and C are all disorders that are now listed under the diagnosis autism spectrum disorder. The description listed represents characteristics of ASD (D).

140
Q

Jerome Kagan discovered that:
Select one:

A.
Infants require a period of separation and individuation from their mothers.

B.
Goodness of fit is crucial to a child’s development.

C.
An infant’s temperament falls into one of three categories: Easy, difficult, or slow to warm up.

D.
An infant’s level of inhibition tends to persist until he or she reaches the age of eight.

A

The correct answer is D.

Jerome Kagan is a key pioneer in developmental psychology and is known for research that showed that temperament is quite stable over time, in that certain behaviors in infancy are predictive of other behaviors in adolescence. Kagan found that the infant inhibition trait tends to persist until the child reaches the age of eight. Goodness of fit and the stability of a child’s temperament was discovered by Thomas and Chess, which makes answers B and C correct. Answer A is a distractor.

141
Q

Which of the following statements regarding vocational interest inventories is not supported by the evidence?
Select one:

A.
They do not predict occupational choice very well.

B.
Test-takers could, if they wished, manipulate the results.

C.
They do not predict future job satisfaction well.

D.
They do not predict future job performance very well.

A

The correct answer is A.

Evidence suggests that Interest inventories are good at predicting occupational choice. However, the results for prediction of job satisfaction and job performance are not as good (cognitive ability measures and general personality measures make better predictions), which makes C incorrect. Speculation is that once an occupation is chosen and entered, the forces that made that job preferred over others are overwhelmed by more immediate pressures. For example, one might choose to be a police officer, but satisfaction with that choice, and good performance as an officer, depend not on the choice, but on workload, stress, home-life balance, and so on (which makes D incorrect). Interest inventories are constructed in such a way that their purpose is often transparent. The assumption is that test-takers will see no advantage in manipulating their scores toward a particular outcome. However, they are vulnerable to such manipulation, which makes B incorrect.

142
Q

Behavior modification is NOT used in the treatment of which of the following?
Select one:

A.
Conduct disorder

B.
Tourette’s syndrome

C.
Pain management

D.
Functional neurological symptom disorder (conversion disorder)

A

The correct answer is D.

Behavior modification addresses conduct disorder (answer A) when it involves the family, and Tourette’s syndrome (answer B), in addition to awareness training, relaxation training, cognitive therapy, and pain management (answer C), related to increasing a healthy lifestyle. Behavior modification is not used directly in conversion disorder (answer D).

143
Q

The General Principles of the APA Ethics Code are best described as:
Select one:

A.
Laws.

B.
Enforceable standards.

C.
Aspirational goals.

D.
Professional absolutes.

A

The correct answer is C.

The general principles are broader and less concrete compared to the ethical standards. They are described in the text as “aspirational goals.” Nothing in the ethical code constitutes law (answer A). Enforceable standards are enforced by law; general principles are broader than this, which makes answer B incorrect. Answer D is a distractor.

144
Q

Approximately what percentage of college students meet criteria for substance abuse or dependence?
Select one:

A.
75%

B.
50%

C.
29%

D.
20%

A

The correct answer is D.

The most common drug used by adolescents is alcohol, which is a major problem on most college campuses. 20% of college students meet criteria for substance abuse or dependence. Answers A, B, and C are incorrect percentages.

145
Q

Which of the following is NOT a mixed method design?
Select one:

A.
Embedded

B.
Descriptive

C.
Triangulation

D.
Explanatory

A

The correct answer is B.

Embedded (answer A), triangulation (answer C), and explanatory (answer D) are all types of mixed method designs, but descriptive is not. The fourth mixed model design is exploratory.

146
Q

Ted believes that poor people do not deserve aid because they are lazy and their current predicament was surely caused by their poor lifestyle choices. Which of the following statements about Ted is most likely true?
Select one:

A.
Ted would score high on the “belief in a just world” scale.

B.
Ted is a very compassionate person.

C.
Ted would score high on the “altruism” scale.

D.
Ted holds liberal views on the issue of poverty.

A

The correct answer is A.

Ted appears to be a perfect candidate for the “belief in a just world” scale. If Ted were a compassionate person, scored high on the altruism scale, or held liberal views on poverty, he would be more inclined to attempt to help poor people in their predicaments, which makes answers B, C, and D incorrect.

147
Q

Studies have shown that in explaining success and failure on an intelligence test, college students are likely to explain others’ poor performance in terms of their ability, whereas they explain their own poor performance in terms of difficulty of the test items. This illustrates:
Select one:

A.
Projective identification.

B.
Self-perception.

C.
Fundamental attribution error.

D.
Actor-observer bias.

A

The correct answer is D.

This situation with the intelligence test attribution illustrates actor-observer bias. Actor-observer bias analyzes the relationship and perceptions between the actor and the observer within a given context or situation, accounting for our ability to self-justify our behaviors. Self-perception (answer B) purports that one gains knowledge about oneself using a similar process to that through which one learns about others. The individual first analyzes observed self-behaviors within a series of contexts. The information gained through various observations allows one to attribute causality of one’s own behavior. Fundamental attribution error (answer C) describes the cognitive tendency where one attributes the observed behavior of others to internal (dispositional) factors, overemphasizing internal factors while failing to consider situational (external) influences, which is close but still makes answer C incorrect. Projective identification (answer A) is a defense mechanism of the ego in which one behaves in such a way toward a person that it produces the emotions, ideas, and behaviors projected onto that person.

148
Q

While riding his bike to school, Tom was hit by a car and suffered a traumatic brain injury, accompanied by temporary amnesia for events that occurred a day prior to the accident and a few hours after the accident. Which of the following memories is most likely to return first?
Select one:

A.
What he ate for breakfast the morning of the accident.

B.
What questions the paramedics asked him at the scene.

C.
His ride to school the morning of the accident.

D.
What he did the evening before the accident.

A

The correct answer is D.

Temporary amnesia can occur as a result of traumatic brain injury. It can involve both anterograde and retrograde amnesia. When the memories return, they typically return in a temporally graded fashion, with memories for events that occurred further away from the traumatic event returning first. Therefore, in this example, what Tom did the evening before the accident is likely to be the first memory that returns. Answer A is incorrect, as breakfast on the morning of the accident is more recent than the evening before the accident, and thus the memory for this event would return later. Answer B is incorrect, as the conversation with the paramedics took place right after the accident, so it is likely to return close to last. Answer C is also incorrect, as the ride to school the morning before the accident is closer to the event than is the evening before the accident. Thus, this memory will return after memories for the evening preceding the accident.

149
Q

Which of the following statements regarding stepwise regression is incorrect?
Select one:

A.
Stepwise regression is a combination of forward and backward regression analysis.

B.
Stepwise regression has an increased probability of making a type I error.

C.
Stepwise methods are not affected by multicollinearity.

D.
Stepwise methods can yield R-squared estimates which are too high.

A

The correct answer is C.

Multicollinearity is of particular concern with stepwise methods, more so than when compared with other regression methods. Answers A, B, and D are all true of stepwise regression.

150
Q

A boy hears a loud noise. He thinks it must be a gunshot. His heart races, and he feels frightened. This example, where a thought precedes the emotional and physiological response, is an example of a theory proposed by:
Select one:

A.
James and Lange.

B.
Lazarus.

C.
Yerkes and Dodson.

D.
Cannon and Bard.

A

The correct answer is B.

Lazarus’s theory of emotion states that the cognitive assessment of one’s circumstance precedes the physiological response and the emotion. Answer A is incorrect because the James-Lange theory of emotion states that the identification of emotion follows a physiological event but does not address the role of cognition in processing emotional information. Answer C is incorrect because the Yerkes-Dodson theory is about the relationship between arousal and performance and does not address the role of cognition in emotional processing. Answer D is incorrect because the Cannon-Bard theory of emotion states that a physiological response follows the experience of emotion but does not address the role of cognition in processing emotional information.

151
Q

FP involves the application of scientific psychological principles to judicial matters. A small percentage of forensic criminal psychologists may work as criminal profilers for the police and the Federal Bureau of Investigation. However, most criminal forensic psychologists are more likely to be involved with:
Select one:

A.
Working in criminal, civil, and correctional law.

B.
Working in correctional and psychiatric facilities.

C.
Evaluating competency to stand trial.

D.
Civil lawsuits involving product liability.

A

The correct answer is C.

Most criminal forensic psychologists are involved with the assessment of competency to stand trial and the defendant’s mental state at the time of the offense (insanity); they may also serve as expert witnesses or evaluate potential jury members. Answer A is incorrect. The question specifically asks about “criminal” forensic psychologists; this is an example of the need to really read and understand the question during the EPPP. Answer B is also incorrect, as forensic correctional psychologists, rather than criminal forensic psychologists, work in the correctional field. Answer D is incorrect, as this is a clear reference to a forensic civil issue rather than a criminal issue. Therefore, it would involve a civil, rather than criminal, forensic psychologist.

152
Q

Neuroleptics may be administered to treat symptoms of __________, which are associated with an__________ of __________.
Select one:

A.
depression; decrease; serotonin

B.
anxiety; increase; norepinephrine

C.
neuralgia; decrease; GABA

D.
psychosis; increase; dopamine

A

The correct answer is D.

Neuroleptics may be administered to treat symptoms of psychosis, which are associated with an increase of dopamine due to blocking the D2 dopamine receptors.

153
Q

Depolarization occurs when sodium channels within the cell membrane:
Select one:

A.
Open and allow positively charged sodium ions to enter the cell.

B.
Open and allow positively charged sodium ions to exit the cell.

C.
Close and keep positively charged sodium ions from entering the cell.

D.
Close and keep positively charged sodium ions from exiting the cell.

A

The correct answer is A.

Depolarization occurs when the threshold of excitation is reached, and the sodium (Na+) channels within the cellular membrane open, allowing positively charged sodium ions to enter the cell.

154
Q

Michael is revamping the performance appraisal system for his department. He has assigned managers to identify key behaviors that contribute to effective and ineffective performance of the key job duties of their subordinates’ jobs. Performance of these behaviors is then recorded, and the frequencies of performance of each are tallied. The resulting information is used to provide feedback that can be used to help the employee increase the frequency of good behaviors while reducing bad behaviors. The method that Michael and his staff are using is called:
Select one:

A.
Employee self-reports.

B.
Enhanced performance indicators.

C.
Judgmental anchors.

D.
Critical incidents.

A

The correct answer is D.

Critical incidents are behaviors that have been identified by subject matter experts (SMEs), typically job incumbents and supervisors, as important to successful or unsuccessful job performance. In the critical incident technique, instances of performance of the behaviors are noted and tallied, and corrective training and counseling put in place. They are not generated by the employees themselves (Answer A), and so they are unrelated to self-ratings of any kind. They are as often associated with unsuccessful performance as with successful performance, so they could not be called enhanced performance indicators (Answer B). In the behaviorally anchored rating scales (BARS) system, they are used as judgmental anchors (Answer C); however, Michael is using the critical incidents technique, not the BARS system.

155
Q

The average of a set of scores is also known as:
Select one:

A.
The mean.

B.
The median.

C.
The mode.

D.
Pearson’s r.

A

The correct answer is A.

The mean is the average value of a set of scores. The mode (answer C) is the most common value, and the median (answer B) is the value in the middle. Pearson’s r (answer D) is a measure of the relationship between two variables.

156
Q

Children who show strong attachment to primary caregivers often express separation anxiety as:
Select one:

A.
hiding under the bed.

B.
clinging to the caregiver.

C.
no reaction at all.

D.
breaking toys.

A

The correct answer is B.

Separation anxiety is caused by separation from a primary caretaker and begins at about six to eight months of age. Children experience separation anxiety when primary caregivers, or parents, leave their child with another person, such as a babysitter, family member, or a stranger at a day care center. Some children may express panic and anxiety when caregivers temporarily leave the room, or show signs that demonstrate departure, such as putting on a jacket or grasping car keys.

157
Q

The therapeutic technique called __________ is based on the experimental procedure called counterconditioning.
Select one:

A.
systematic desensitization

B.
biofeedback

C.
latent learning

D.
autogenic training

A

The correct answer is A.

Counterconditioning is a Pavlovian conditioning technique that involves conditioning an individual to respond to a stimulus in a way that is incompatible with a response that has already been conditioned to that stimulus. The technique of systematic desensitization (answer A) is a variant of counterconditioning. Biofeedback (answer B) involves monitoring a bodily function (such as breathing, heart rate, or blood pressure) and altering the function through relaxation, imagery, or some other mental activity. Latent learning (answer C) is a form of learning that is not immediately expressed in an overt response; it occurs without obvious reinforcement to be applied later. Autogenic training (answer D) is a treatment for anxiety that involves regular practice of relaxation techniques.

158
Q

You are invited by Paul to a party where he introduces you to a friend of his. The three of you chat for a few minutes and then Paul leaves, leaving you and the friend alone. You chat for several more minutes and part ways. After the party, Paul asks you what you thought of his friend. You tell him that you thought his friend was attractive and engaging and that you generally liked his friend. Paul relays this message to his friend. The next time you meet Paul’s friend, the friend is very warm, smiles a lot, discloses more information about him or herself, and otherwise is very friendly. The best explanation for this behavior is:
Select one:

A.
The reciprocity principle.

B.
Matching theory.

C.
The similarity hypothesis.

D.
Congruency theory.

A

The correct answer is A.

By telling Paul that you think that his friend is attractive and engaging, you have set up a perfect opportunity for the reciprocity principle to come into effect, as long as Paul relays your comments to his friend. Answer C is incorrect because there is no such thing as congruency theory. Answer B is incorrect because matching theory describes how people of a given level of physical attractiveness will seek out partners of a roughly similar level of attraction, and looks were not mentioned in this scenario. Answer D is a distractor.

159
Q

For a few years, Tim has been working as a nurse in the fast-paced environment of the city’s hospital ER. Tim’s schedule varies as he is on call. While on call, he can get paged at any time of day or night. Tim moved to the town he lives in because of the schedule his job demands. At this point in time, he has not had the chance to develop effective supports. He is also single and lives alone in his apartment. He does not do much when he is not at work other than to recuperate from the intense activities at the hospital. He does not exercise or engage in other stress-reduction activities. He normally does not go out to shows, music concerts, art museums, or other engagements. He spends the weekends getting ready to return to work. Lately, Tim has been exhibiting more frequent absenteeism, having lost the initial interest and motivation he derived from his job. He feels that people he works with are stressing him out. He finds it very demanding to perform his duties, as he feels ineffective and overall “spread thin.” What is likely going on with Tim?
Select one:

A.
Leaving his hometown, family, and friends has finally caught up with him, causing him to become depressed

B.
He is experiencing symptoms of job burnout

C.
Tim has developed a mixed depression and anxiety disorder due to his unpredictable schedule

D.
The lack of exercise and ability to relax and replenish when at home is negatively affecting his job life

A

The correct answer is B.

Burnout is severe psychological strain that occurs when an individual is continuously exposed to chronic stressors and has not developed the coping mechanisms to deal with them. Burnout is characterized by a state of depleted energy reserves, lowered resistance to illness, increased dissatisfaction and pessimism, increased inefficiency, higher rates of absenteeism, and turnover. Researchers have identified three major components of burnout: Emotional exhaustion (feeling emotionally drained by work demands), depersonalization and cynicism (typified by treating others as objects), and feelings of ineffectiveness or loss of self-efficacy. Answer A is a good interpretation for what is going on but is not the best choice here, as are answers C and D. Although all of these indicators could be included in causing job burnout, these answers are not as specific as Answer B.

160
Q

__________ is the personality dimension from the Big Five that is most highly correlated with success across virtually all occupations.
Select one:

A.
Emotional stability

B.
Extraversion

C.
Agreeableness

D.
Conscientiousness

A

The correct answer is D.

Researchers have successfully linked the Big Five to important work outcomes. Meta-analyses have shown, in particular, that conscientiousness is related to job performance and, more broadly, to success across virtually all occupations (Landy et al., 2004). Emotional stability (Answer A), agreeableness (Answer C), and extraversion (Answer B) predict success in some occupations, but not all.

161
Q

You have heard that your neighbor, a new immigrant from Southeast Asia, has been struggling to earn enough money to eat. As you are cooking dinner the following night, you prepare extra for your neighbor and bring a plate over to her. She is very grateful and accepts the gift. To your surprise, you find a small gift basket of goodies at your front door the very next morning. The most reasonable explanation for your neighbor’s behavior is:
Select one:

A.
The reciprocity principle.

B.
The reciprocal principle.

C.
The complementarity hypothesis.

D.
The similarity hypothesis.

A

The correct answer is A.

Your neighbor’s behavior is an example of reciprocity. Research has been carried out on the reciprocity principle and gift-giving. The reciprocity principle suggests that when one person receives a gift, the receiver will return the favor to the giver. Answer D is incorrect because the similarity hypothesis describes how people tend to like and love others with similar opinions, attitudes, values, and looks; in this particular story that is not what occurred. The reciprocal principle and the complementarity hypothesis are both distractors, which makes answers B and C incorrect.

162
Q

Dr. Quiroz conducted a competency evaluation for William, a 17-year-old who was accused of dealing drugs, at the Jacksonville Federal Court. Which of the following statements is most likely to be found in her report to the court?
Select one:

A.
“During the interview, William admitted to enjoying his ‘exciting’ life as a drug dealer and remarked that he plans to continue this lifestyle when he is released from jail.”

B.
“During the interview, William was not oriented to person, place, or time. He also reported hearing voices several times a week.”

C.
“William refused to waive privilege during this evaluation.”

D.
“In my opinion, William is guilty of the charges. Jail is warranted, as is his being released into his parents’ custody for the probationary period.”

A

The correct answer is B.

The sole issue in a competency evaluation is whether the client can assist in his or her own defense. Answer B relates to his current mental status and is therefore relevant. Answer A is not informative with regard to his competency. Answer C is moot; privilege is automatically waived in this type of evaluation. Answer D is irrelevant to the evaluation, and such a statement would likely lead to a lawsuit from the client or the client’s legal representative.

163
Q

Individuals who experience migraine headaches are more likely than people in the general population to be diagnosed with:
Select one:

A.
Bipolar disorder.

B.
Seizure disorder.

C.
Alzheimer’s disease.

D.
Vascular dementia.

A

The correct answer is B.

Migraine headaches are linked to seizures in some cases. Answer A is a distractor, as bipolar disorder is not specifically associated with headaches. Answers C and D are also incorrect, as they do not have noteworthy comorbidity with migraine headaches.

164
Q

Von Willebrand disease is characterized by which of the following?
Select one:

A.
Ataxia, apraxia, and alexia

B.
Excessive bleeding and difficulty clotting

C.
Excessive non-functional white blood cells

D.
Involuntary movements and dancing gait

A

The correct answer is B.

Von Willebrand disease is the most common inherited bleeding disorder. It is characterized by excessive bleeding and prolonged bleeding times due to difficulty clotting despite adequate blood platelets. Symptoms also include bruising, bleeding gums, and frequent nose bleeds. Ataxia, apraxia, and alexia have to do with a lack of muscle control with involuntary movements, not excessive bleeding, which makes answer A false. Von Willebrand disease does not address excessive non-functional white blood cells but excessive bleeding despite adequate blood platelets, which makes answer C incorrect. Involuntary movements and dancing gait are not involved in Von Willebrand disease, which makes answer D incorrect.

165
Q

According to the Ethics Code, psychologists should establish compensation and billing arrangements with a new client:
Select one:

A.
Prior to beginning therapy.

B.
During the initial therapy session.

C.
Once the psychologist has established a rapport with the client.

D.
As soon as is feasible.

A

The correct answer is D.

Fees should be set as soon as is feasible. Answer A is true in most situations, but in emergency situations it may not be feasible to discuss fees before beginning therapy.

166
Q

The APA Ethics Code permits psychologists to have sexual relationships with former clients when at least two conditions have been met. The first criterion states that at least two years must have passed since the cessation of therapy before the sexual relationship begins. The second criterion states that a sexual relationship may occur:
Select one:

A.
If the client believes that he or she will not need additional therapy in the future.

B.
If the psychologist can document that he or she never made promises regarding the possibility of future relationships during the course of therapy.

C.
If the psychologist saw the client for fewer than five therapy sessions.

D.
Under the most unusual circumstances.

A

The correct answer is D.

Answers A and C are distractors. Answer B is one of the criteria that psychologists must consider in this situation, but there are others. Thus, if answer B is true, this does not provide sufficient reason to pursue a sexual relationship (although it is a necessary condition).

167
Q

Nancy Chodorow developed:
Select one:

A.
An additional aspect of the self-in-relation theory.

B.
The psychoanalytic feminist therapy theory.

C.
An additional aspect of humanistic therapy.

D.
A cognitive model of feminist therapy.

A

The correct answer is B.

Nancy Chodorow is credited with the development of the psychoanalytic feminist therapy theory. She was not the key figure in self-in-relation theory (answer A), nor did she develop additional aspects of humanistic therapy (answer C). Her model focused on mother-daughter and mother-son roles and the early Freudian bisexuality that is developed through one’s relationship with the mother. As such, her model does not address a cognitive model (answer D) but a model involving ego development through maternal relationships.

168
Q

Before providing services to an 11-year-old child, a psychologist should do all of the following EXCEPT:
Select one:

A.
Obtain informed consent from the child’s legal guardian.

B.
Provide the child with an appropriate explanation of the services.

C.
Consider the child’s preferences.

D.
Obtain informed consent from the child.

A

The correct answer is D.

An 11-year-old is incapable of providing informed consent. Nevertheless, an appropriate explanation of services should be provided to the child (answer B), and the child’s wishes regarding services should be considered (answer C). In addition, informed consent must be obtained from the legal guardian (answer A).

169
Q

All of the following are often true of early-maturing girls EXCEPT:
Select one:

A.
They experience more teasing and ridicule from their peers.

B.
They have higher rates of reported symptoms of anxiety and depression.

C.
They tend to socialize with an older peer group.

D.
They are more popular than their prepubescent peers.

A

The correct answer is D.

An early-maturing girl is often less popular than her prepubescent classmates. Symptoms such as depression and anxiety are more likely to be reported by early-maturing girls, especially if they have experienced other psychological problems in childhood. This makes answer B, which is true, incorrect. In addition, early-maturing girls tend to make friends with older peer groups. Because of this, they are more likely to become involved in the things that the older peer group is interested in, including dating, drinking, having sex, and engaging in other risk-taking behaviors. This makes answer C incorrect as well. On the other hand, although later-maturing girls may experience some anxiety while they wait to mature, they often surpass other students on achievement tests. They may also experience body image problems and more conflicts with parents. Answer A is incorrect because it is true that early-maturing girls may experience more teasing and ridicule from their peers.

170
Q

As predicted by the Taylor-Russell selection tables, incremental validity of a new selection procedure with a validity coefficient of .50 is the most useful when:
Select one:

A.
The selection ratio is .95 and the base rate is .20.

B.
The selection ratio is .90 and the base rate is .60.

C.
The selection ratio is .05 and the base rate is .95.

D.
The selection ratio is .05 and the base rate is .60.

A

The correct answer is D.

When the incremental validity of a new selection procedure has a moderate to high validity, it is most useful to have a lower selection ratio and a moderate base rate.

171
Q

The brain damage responsible for Korsakoff’s syndrome is caused by:
Select one:

A.
The toxic effects of alcohol.

B.
A reduction in the supply of blood to a brain area.

C.
An ischemic stroke.

D.
A vitamin deficiency.

A

The correct answer is D.

Korsakoff’s syndrome is a memory disorder that occurs in some individuals after many years of heavy alcohol consumption. It is characterized by both anterograde amnesia and retrograde amnesia. Although Korsakoff’s syndrome occurs almost exclusively in alcohol-dependent individuals, the primary etiological factor is actually a deficiency in the availability and utilization of thiamine (vitamin B1) by brain cells. Alcohol itself does not cause the critical brain damage; therefore, answer A is incorrect. Answers B and C are also incorrect, as an ischemic stroke is a reduction in the supply of blood to a brain area, and although such events can cause brain damage and concomitant memory impairment, they do not produce Korsakoff’s syndrome.

172
Q

Hypervigilance, relaxed but awake, falling asleep, and deep sleep, respectively, correspond with:
Select one:

A.
Alpha, beta, theta, and delta.

B.
Beta, alpha, theta, and delta.

C.
Alpha, beta, delta, and theta.

D.
Beta, alpha, delta, and theta.

A

The correct answer is B.

As this is the correct progression of brain activity (activity gets slower as sleep gets deeper).

173
Q

Which of following statements about gender and depression is most true?
Select one:

A.
Females are twice as likely to experience depression, regardless of age.

B.
In adults, females are twice as likely to experience depression; in children, females and males are equally likely to experience depression.

C.
In adults, females and males are equally likely to experience depression; in children, females are twice as likely to experience depression.

D.
In adults, females are twice as likely to experience depression; in children, females are less likely to experience depression than males.

A

The correct answer is B.

Boys and girls (prior to adolescence) are about equally likely to experience major depression, but during and after adolescence, female prevalence rates are at least twice those of men.

174
Q

Jonathan’s score on his test of verbal fluency falls in the 98th percentile. Dr. Sooz converts his test score into a T-score and types it into the computer. What number would Dr. Sooz need to type for correct data to be entered?
Select one:

A.
50

B.
60

C.
70

D.
80

A

The correct answer is C.

The 98th percentile corresponds to a T-score of 70. A T-score of 50 corresponds to the 50th percentile (answer A). A T-score of 60 corresponds to the 84th percentile (answer B). A T-score of 80 corresponds to the 99th percentile (answer D).

175
Q

Which of the following statements regarding biofeedback is correct?
Select one:

A.
Biofeedback has been shown to be an effective treatment for a number of psychophysiological disorders.

B.
EMGs are the instrument of choice to treat phobias.

C.
Biofeedback is a standard treatment for reducing blood pressure.

D.
GSR training is used to treat headaches and neck pain.

A

The correct answer is A.

Biofeedback has been successfully applied to a number of psychophysiological disorders, such as phobias, headaches, and high blood pressure. EMGs are the treatment of choice for backaches and headaches but are not used for phobias (answer B), which are more likely to respond to the GSR biofeedback approach. While biofeedback has been used to reduce blood pressure (answer C), it is not the standard treatment, as relaxation appears to provide more benefit than biofeedback alone. GSR (answer D) is not used for headaches or neck pain, but it is used in the treatment of anxiety disorders.

176
Q

Mrs. Howell, a 45-year-old woman, has been diagnosed with severe mental retardation and requires full-time care. She was recently evaluated by a neuropsychologist and his findings are being subpoenaed by the courts because her insurance claims that she is malingering, or feigning mental illness. Mrs. Howell’s 25-year-old daughter is her legal guardian. In this case, who would be considered Mrs. Howell’s holder of privilege?
Select one:

A.
Her primary care doctor

B.
Her daughter

C.
The courts

D.
The director of the residential facility where she resides

A

The correct answer is B.

An individual with severe mental retardation would not be legally competent to assert privilege; the holder of privilege is, therefore, Mrs. Howell’s legal guardian. This makes answers A, C, and D incorrect.

177
Q

Some of the specifiers found in persistent depressive disorder (formerly dysthymic disorder) are:
Select one:

A.
Pure dysthymic symptoms, persistent major depressive episode, current episode.

B.
Intermittent major depressive episode, persistent major depressive episode, pure dysthymic syndrome.

C.
Persistent major depressive symptoms, pure dysthymic syndrome, intermittent major depressive episode.

D.
Current episode, pure dysthymic syndrome, intermittent major depressive symptoms.

A

The correct answer is B.

These are some of the specifiers that are found in persistent depressive disorder. There is no such specifier as pure dysthymic symptoms (it is “pure dysthymic syndrome”), making answer A false. There is no such specifier as persistent major depressive symptoms (it is “persistent major depressive episode”), which makes answer C incorrect. There is no specifier for intermittent major depressive symptoms (it is “intermittent major depressive episodes”) in persistent depressive disorder, which makes answer D false.

178
Q

The typical way psychologists study habituation is by measuring levels of:
Select one:

A.
Satiation.

B.
Arousal.

C.
Boredom.

D.
Non-association.

A

The correct answer is B.

During habituation, the repeated stimulus has a progressively weaker effect on the organism. In some cases, this can be observed through changes in physiological measures of arousal, such as heart rate, blood pressure, EEG, galvanic skin response, and others. Satiation (answer A) is related to reinforced behavior, whereas habituation does not involve reinforcement. Boredom (answer C) can be influenced by many factors that do not involve the repeated presentation of a stimulus or a decrease in the strength or likelihood of a response, and an individual’s level of boredom may be unchanged throughout the habituation process. Habituation is a “non-associative” process. Habituation does not involve learning about associations among external stimuli or events, which makes answer D incorrect.

179
Q

If you believe that men are better drivers than women, you will likely notice more traffic violations by women and ignore the same infractions committed by men. This tendency is referred to as:
Select one:

A.
Confirmation bias.

B.
Saliency bias.

C.
Self-fulfilling prophecy bias.

D.
Correspondence bias.

A

The correct answer is A.

What you are attempting to do is to confirm your previous theories about drivers, specifically about drivers with regards to gender, which makes what you are doing an example of confirmation bias. Confirmation bias is the tendency to search for information that confirms one’s preconceptions. Saliency bias is when a vivid or salient feature in a situation is viewed as the cause of a behavior, which makes answer B incorrect. Self-fulfilling prophecy is the tendency for people to act in a way that is consistent with their expectations about an event or behavior, thereby increasing the likelihood that the event or behavior will occur, which makes answer C incorrect. Self-fulfilling prophecy is similar but does not refer only to noticing things as confirmation bias does. Correspondence bias is the fundamental attribution error, where a person has the tendency to overestimate the extent to which people’s behavior is due to internal dispositional factors and to underestimate the role of situational factors, which makes answer D incorrect.

180
Q

Patients with memory loss following damage to the medial-temporal lobe region display impairments in:
Select one:

A.
Short-term memory.

B.
Procedural memory.

C.
Implicit memory.

D.
Explicit memory.

A

The correct answer is D.

Medial-temporal lobe damage can produce severe impairment in anterograde long-term memory, or the ability to form new long-term memories. However, not all types of long-term memory are impaired, and those that are involve explicit memory (answer D). Amnesic patients have difficulty consciously recollecting personal experiences and factual information, and explicit memory is the term used to refer to these abilities. Implicit memory (memory without conscious awareness) is not impaired in amnesia, and procedural memory (answer B) is a type of implicit memory (answer C). Short-term memory (answer A) abilities are also spared in patients with amnesia resulting from medial-temporal lobe damage.

181
Q

Vestibule training is most useful when:
Select one:

A.
Training costs need to be cut.

B.
Teaching basic skills.

C.
Teaching complex skills.

D.
On-the-job training would be problematic.

A

The correct answer is D.

Vestibule training provides simulated experiences related to the work setting, such as mock assembly or flight simulators. This training is used when errors are high and repeated practice is necessary to complete the objective at hand, or when on-the-job training could be too dangerous (Answer D; for example, the use of flight simulators for pilots). Vestibule training can be expensive; thus, Answer A is not correct. Vestibule training can involve both basic (Answer B) and complex (Answer C) skills, as can the other types of training. The key difference is the need to practice before being on the job.

182
Q

According to the DSM-5, in which of the following disorders are individuals most likely to develop comorbid obsessive-compulsive disorder?
Select one:

A.
Asperger’s disorder

B.
dependent personality disorder

C.
Tourette’s disorder

D.
hypochondriasis

A

The correct answer is C.

Tourette’s disorder has a particularly strong overlap or comorbidity with OCD. Answers A, B, and D are incorrect, as these conditions have some amount of comorbidity, but not as strong.

183
Q

A psychologist is considered licensed when:
Select one:

A.
He or she meets all requirements set by his or her state or province.

B.
He or she passes the EPPP and completes needed internship requirements.

C.
He or she completes a doctorate of psychology degree and passes the EPPP.

D.
He or she has taken the EPPP.

A

The correct answer is A.

For most all states, licensure requires not only taking the EPPP (answer D) but also fulfilling other requirements determined by the state/province. Most states have a requirement of internship (answer B) and a doctorate degree in psychology (answer C), but these differ per state, making A the best answer.

184
Q

Emily has been seeing Dr. Spocker for four years. She recently decided that all of her therapy goals have been reached and she would like to stop treatment. Ethically speaking, what should Dr. Spocker do?
Select one:

A.
Encourage her to continue therapy

B.
Terminate as soon as feasible

C.
Provide a few sessions of pretermination therapy and then terminate

D.
Encourage the client to devote a few sessions to investigate the real motivation underlying her thoughts and decision

A

The correct answer is C.

The client’s decision should be accepted, and given the length of time for treatment, pretermination therapy would be appropriate to summarize and process the end of a long therapeutic relationship. Answer A would be unethical, given the situation and the achievement of the client’s goals. Answer B is incorrect, as this would be too rushed, given that the treatment lasted for four years. Answer D is incorrect and unethical, as there is no valid reason why the therapist should not accept the client’s decision and slow down the therapeutic movement at a time when it should be evaluated and energized, with termination being the new objective. It is clearly time to process termination issues and launch the client into her new life.

185
Q

The main difference between Maslow’s hierarchy of needs and ERG theory is:
Select one:

A.
Maslow’s theory contains three levels of needs that must be met in a given order, while ERG theory contains three levels of needs that can be met in any order.

B.
ERG theory is a flexible theory of human motivation where an individual can have growth needs as a priority over existence needs, while according to Maslow’s hierarchy of needs, an individual must have certain lower-level needs met before he or she can focus on higher-level needs.

C.
Maslow’s hierarchy of needs is flexible, while ERG theory is continuous.

D.
ERG theory considers money to be a higher-order need, while Maslow’s theory considers it a lower-order need.

A

The correct answer is B.

The main difference between the two theories involves how in ERG theory, a person can “regress” to a lower level called the “frustration-regression principle,” and that what level is being regressed to is not as important as the act of regressing, whereas Maslow’s hierarchy must have particular lower-level needs met before higher-level needs can be focused on, which makes answer B correct. Maslow’s hierarchy is not as flexible as ERG theory, which makes answer C incorrect. ERG theory does not consider money to be a higher-order need, which makes answer D correct. Maslow’s theory is not the one that contains three levels of needs but rather it contains five different needs, which makes answer A incorrect.

186
Q

Dr. Hyde has become interested in one of his supervisees and believes that she likes him too. He decides to ask her out on a date. This is considered:
Select one:

A.
Unethical because it would be considered exploitative.

B.
Unethical because it would show favor toward her over his other supervisees.

C.
Ethical as long as she is not coerced into the date.

D.
Ethical as long as both adults are consenting and acknowledge the inherent power differential.

A

The correct answer is A.

Given the power differential between supervisor and supervisee, there is no such thing as true mutual consent in such situations. Answer D is incorrect, as multiple relationships are unethical when they exploit an inherent power differential. Answer C is incorrect; given the power differential, coercion is an unavoidable factor in this request. Answer B is true and a negative consequence of this behavior, yet it is not the critical issue in making the relationship unethical.

187
Q
Which neurotransmitter(s) is/are most involved in the early and later stages of Alzheimer's disease?
Select one:

A.
Dopamine in the early and later stages

B.
Serotonin and norepinephrine in the early and later stages

C.
GABA and glutamate in the early stages; acetylcholine and dopamine in the later stages

D.
Acetylcholine in the early stages; serotonin, norepinephrine, and glutamate in the later stages

A

The correct answer is D.

Alzheimer’s disease is characterized by progressive memory loss and involves disruption of acetylcholine neurotransmitters in the early stages and serotonin, norepinephrine, and glutamate in the later stages. Parkinson’s disease is related to the disruption of dopamine (answer A). Depression is related to the disruption of serotonin and norepinephrine (answer B). Answer C combines a variety of inhibitory and excitatory neurotransmitters, and this early- and late-stage pattern is not related to any specific brain disease.

188
Q

For an accurate diagnosis of mania, symptoms must:
Select one:

A.
Last at least one week unless the person is hospitalized.

B.
Last at least four days and include a state of intense elation or irritability.

C.
Last at least two weeks unless they are cyclical or recurrent.

D.
Last at least one week and include rapid speech.

A

The correct answer is A.

According to the DSM-5, a diagnosis of mania will include manic symptoms lasting a minimum of one week, unless the person’s symptoms are so severe that he or she is hospitalized. In this situation, the person may receive the diagnosis even if symptoms were present for less than a week. Mania does include a state of intense elation or irritability, but four days of symptoms (answer B) would not warrant a diagnosis of mania unless the person was hospitalized. Manic symptoms may cycle with depressive symptoms and may be recurrent but are not required for a diagnosis; in addition, symptoms are not required to last for two weeks (answer C). Rapid speech is a common symptom of mania (answer D) but is not required for a diagnosis; therefore, answer A is the preferred answer.

189
Q

You are seeing a 35-year-old male client for court-ordered therapy related to his drug use. The client reveals that he molested a 10-year-old neighbor 15 years earlier. Although the client insists that he has not harmed any children since, his discussions regarding his new girlfriend’s 8-year-old daughter lead you to suspect that he may currently be molesting this girl. Your most ethical course of action would be to:
Select one:

A.
Refer the 8-year-old girl for a sexual abuse assessment.

B.
Continue seeing the client, being particularly aware of any statements he makes that provide more concrete evidence that he is molesting the girl.

C.
Report the suspected molestation to a child protective services agency or the police.
Correct

D.
Warn the new girlfriend about the risk.

A

The correct answer is C.

In some states, the revelation of the prior act of child abuse is reportable in itself; in other states, the knowledge of the past act is reportable if the adult has present responsibilities involving children. However, regardless of state law, the prior act, along with one’s clinical judgment regarding current risk, is very likely to provide “reasonable cause” to suspect the occurrence of present child abuse. Thus, mandated reporting is warranted. Answers A, B, and D do not fulfill the mandated reporting requirement–the local child protective agency must be contacted.

190
Q

Some of the characteristics of major neurocognitive disorder with Lewy bodies (NCDLB) are:
Select one:

A.
Rapid cognitive impairment, recurrent complex visual hallucinations, and REM sleep behavior disorder.

B.
Schizophrenia, depression, and delusions.

C.
Recurrent complex visual hallucinations, REM sleep behavior disorder, and delusions.

D.
REM sleep behavior disorder, hallucinations in other sensory modalities, depression, and paranoia.

A

The correct answer is C.

Hallucinations, REM sleep disorders, and delusions may be associated with NCD with Lewy bodies. Rapid cognitive impairment is not part of major neurocognitive disorder with Lewy bodies, making answer A incorrect. Answer B is incorrect because schizophrenia is not an associated feature in NCD with Lewy bodies (NCDLB). Answer D is incorrect because paranoia is not an associated feature in NCD with Lewy bodies.

191
Q

Reactance refers to:
Select one:

A.
the tendency to blame others for our mistakes.

B.
the tendency to do the exact opposite of what is being requested.

C.
the tendency to revert to a childlike state when facing stress.

D.
the tendency to forget an event due to the intensity of the stress.

A

The correct answer is B.

Reactance is defined as a term for an emotional reaction in which people perform, or believe, the exact opposite of what is being requested, due to a need to protect a personal sense of freedom and autonomy, which makes Answer B correct. Reactance does occur in children but does not specifically refer to the tendency to revert to a childlike state when facing stress, which makes Answer C incorrect. The tendency to blame others for our mistakes is more of an attribution error as opposed to reactance, which makes Answer A incorrect. The tendency to “forget” an event due to the intensity of the stress is not a reactance, which makes Answer D incorrect.

192
Q

Maria believes that gender identity development is formed based on the different messages children receive regarding their behavior. Maria is ascribing to which theory of gender identity development?
Select one:

A.
Social cognitive theory

B.
Psychoanalytic theory

C.
Gender schema theory

D.
Cognitive developmental theory

A

The correct answer is A.

The social cognitive theory contends that males and females are treated differently based on gender and receive rewards or punishment related to gender-appropriate or gender-inappropriate behaviors. Psychoanalytic theory (answer B) emphasizes the concepts of inner-psychic conflicts such as the Oedipal conflict and penis envy. Gender schema theory (answer C) argues that gender identity is the result of an individual’s internal motivation to conform to gender based on sociocultural standards and stereotypes. Cognitive development theory (answer D) emphasizes that gender development occurs through stages of mental development and cognitive processing.

193
Q

The phenomenon characterized by restlessness of thought and movement is referred to as:
Select one:

A.
Akinesia.

B.
Akathisia.

C.
Dystonia.

D.
Avolition.

A

The correct answer is B.

Akathisia refers to a sense of distress and restlessness. It is the most common side effect of antipsychotic medications. Answer A is incorrect, as akinesia refers to the absence or poverty of movement. Answer C is incorrect, as dystonia refers to a disorder that entails repetitive movements and/or abnormal postures; however, like akathisia, it is an extrapyramidal side effect. Answer D is incorrect, as avolition is a lack of motivation or desire and is observed in schizophrenia as part of the negative symptoms.

194
Q

Symptoms of alcohol-induced neurocognitive disorder include all of the following EXCEPT:
Select one:

A.
Apraxia.

B.
Agnosia.

C.
Aphasia.

D.
Executive dysfunction.

A

The correct answer is C.

Aphasia is not a symptom likely to occur in the context of alcohol-induced neurocognitive disorder (previously known as alcohol-induced persisting dementia), whereas apraxia (answer A), agnosia (answer B), and executive function deficits (answer D) are.

195
Q

In computerized adaptive testing, if an examinee gets the very first question (of intermediate difficulty) correct, then the next question should be:
Select one:

A.
Easier.

B.
Harder.

C.
Also intermediate.

D.
Randomized.

A

The correct answer is B.

As each item is selected based on the test taker’s ability to arrive at the correct answer. This means that the next item should be more difficult if the test taker provides the correct response. A, C, and D are incorrect.

196
Q

According to Standard 8.02 of the APA Ethics Code, when obtaining informed consent for research, psychologists should provide prospective participants with information about:
Select one:

A.
Prospective research benefits.

B.
Deceptive measures that will be used in the study.

C.
The researchers’ hypotheses.

D.
Previous research conducted by the psychologists.

A

The correct answer is A.

Informed consent should involve potential research benefits. Information about deceptive measures (answer B) and hypotheses (answer C) is not presented prior to participation; therefore, it is not part of the informed consent process. Information about previous research conducted by the psychologists (answer D) is not a part of the requirements for informed consent either.

197
Q

Stress inoculation is most appropriate with individuals suffering from:
Select one:

A.
Impulse control.

B.
Depression.

C.
Bipolar disorder.

D.
Phobias.

A

The correct answer is D.

Stress inoculation training is based on the premise that bolstering one’s coping responses to milder stressors improves coping skills for more severe stressors; therefore, it can be applied to anxiety disorders such as phobias (answer D). As the coping skills in stress inoculation training are focused on the management of stress, it is less suited for impulse control (answer A). While depression (answer B) and bipolar disorder (answer C) are both affected by stress, they do not respond as well to this type of exposure or coping training.

198
Q

The male-to-female ratio for children and adolescents with ADHD is approximately:
Select one:

A.
5:1.

B.
2:1.

C.
4:1.

D.
1:1.

A

The correct answer is B.

The gender ratio is approximately 2:1 (though it may be higher in clinical settings, where comorbidity with ODD/CD is high). Answers A, C, and D are not gender ratios that are approximate with ADHD, which makes these answers incorrect.

199
Q

According to the James-Lange theory, a person who is undergoing implosive therapy would:
Select one:

A.
Experience no benefit.

B.
Experience moderate benefit.

C.
Have a strong emotional response to the treatment.

D.
Show signs of regression.

A

The correct answer is A.

The James-Lange theory states that bodily responses to arousing stimuli precede subjective experience of emotions (answer A) including fear. The theory does not make a clear prediction about the potential benefits of implosive therapy (answers B and C), nor does it predict that regression would be a consequence (answer D).

200
Q

According to __________, emotions are the result of physiological changes such as muscle tension, increased heart rate, and sweating.
Select one:

A.
Walter Cannon

B.
Stanley Schachter

C.
Jerome Singer

D.
William James

A

The correct answer is D.

William James posited (at the same time as Carl Lange) that emotions are the result of physiological changes. This contrasts Cannon-Bard theory (answer A) that emotions are the cause of physiological changes. Schachter (answer B) and Singer (answer C) are best known for their two-factor theory of emotion, stating that people experience an event and then cognitively appraise physiological arousal to determine what they are feeling.

201
Q

HIPAA privacy laws are applicable when dealing with:
Select one:

A.
Any health care organization.

B.
Any third party.

C.
Any entity that transmits information electronically.

D.
Court-ordered evaluations.

A

The correct answer is C.

HIPAA does not apply to all agencies, as the electronic transmission of information triggers the privacy rule. Any third party and court-ordered evaluations are things that HIPAA addresses, which makes answers B and D incorrect. Answer A is incorrect, as it is incomplete and too vague.

202
Q

Myelogenous leukemia is to lymphocytic leukemia as:
Select one:

A.
Red blood cells are to white blood cells.

B.
Blood cells are to lymphocytes.

C.
Bone marrow is to blood.

D.
Bone marrow is to the spleen.

A

The correct answer is B.

Myelogenous leukemia forms in bone marrow cells that are precursors of red blood cells, platelets, and some white blood cells. Lymphocytic leukemia forms in bone marrow cells that are designated to mature into lymphocytes. Lymphocytic leukemia affects lymphocytes, not white blood cells or blood per se (answers A and C). Lymphocytic leukemia may affect the spleen in some cases, but not all (answer D).

203
Q

Which of the following statements would Locke and other empiricists disagree with?
Select one:

A.
Humans would exist in perfect harmony with one another in the absence of negative environmental factors.

B.
Humans are born as a blank slate.

C.
Human knowledge is gained through direct contact with the physical environment.

D.
There are primary and secondary qualities to the physical environment.

A

The correct answer is A.

Lockean psychology, or empiricism, argues that people are born as a tabula rasa (blank slate; answer B), knowledge is learned through accidental associations via interaction with the physical world (answer C), and sensations are experienced through primary and secondary qualities of the physical environment (answer D). A belief that all people are inherently good and this goodness is affected by only negative environmental factors is a basic tenet of humanistic psychology.

204
Q

If a student obtains a t score of 70, his or her score is in:
Select one:

A.
The 50th percentile.

B.
The 70th percentile.

C.
The 84th percentile.

D.
The 98th percentile.

A

The correct answer is D.

A t score of 70 reflects a raw score two standard deviations above the mean (t scores have a mean of 50 and a standard deviation of 10), as does the 98th percentile (a score above 98% of the scores in a distribution, or two standard deviations above the mean).

205
Q

Which of the following tends to minimize social loafing?
Select one:

A.
Increasing group size

B.
Increasing individual accountability

C.
Increasing group cohesion

D.
Reducing individual visibility

A

The correct answer is B.

When group or team members feel more accountable for the group’s outcomes, and thus also their own outcomes, they are likely to expend more effort. Increasing group size (Answer A), cohesion (Answer C), or individual visibility (Answer D) tends to make members feel less individually accountable.

206
Q

__________ memory is made up of three components: The central executive, phonological loop, and visuospatial scratchpad.
Select one:

A.
Short-term

B.
Working

C.
Episodic

D.
Sensory

A

The correct answer is B.

There are three components that comprise working memory: The central executive, phonological loop, and visuospatial scratchpad. The central executive is the component of working memory that distributes and directs attention to various tasks that make use of the information being held in working memory. The auditory portion of working memory is called the phonological loop. The phonological loop retains verbal information by continuously repeating it over a short period of time. The visual portion of working memory is called the visuospatial scratchpad, and it manipulates and retains visual, imaginal, and spatial information by means of rehearsal. Answer A is incorrect, as although the concept of working memory subsumes the older concept of short-term memory, the older concept does not include multiple components. Episodic memory (answer C) is memory for personal experiences or events, and sensory memory (answer D) is the impression that remains for a brief moment after the termination of a sensory stimulus. Neither theories of episodic memory nor theories of sensory memory include the components listed in the question.

207
Q

Taste aversion learning is considered to be a type of:
Select one:

A.
Survival mechanism.

B.
Covert sensitization.

C.
Habituation.

D.
Negative reinforcement.

A

The correct answer is A.

Taste aversion learning occurs when the taste of a relatively unfamiliar food is associated with gastrointestinal discomfort or illness. This leads to an enduring aversion to that particular taste. The ability to develop a taste aversion is considered an adaptive trait or survival mechanism (answer A) that evolved in a wide range of species as a mechanism through which to avoid being harmed by poisonous substances in the environment. Covert sensitization (answer B) is a technique in behavior therapy in which the client is trained to associate an undesired behavior with aversive or unpleasant stimuli through the use of imagery. Habituation (answer C) involves a reduction in the intensity of a response to an innocuous stimulus, which occurs when that stimulus is repeatedly presented. Negative reinforcement (answer D) is an operant conditioning procedure in which an unpleasant stimulus is removed, contingent on the performance of a particular behavior in order to increase the occurrence of that behavior.

208
Q

Which statement best describes the Zeigarnik effect?
Select one:

A.
People forget uncompleted or interrupted tasks more easily than completed ones.

B.
People will report that a stationary light in a dark room appears to move if stared at long enough.

C.
People remember uncompleted or interrupted tasks better than completed ones.

D.
People will give high accuracy ratings to generic character descriptions that are supposedly tailored specifically to them.

A

The correct answer is C.

The Zeigarnik effect refers to our tendency to remember uncompleted tasks better than completed tasks. Therefore, answer A has the phenomenon backwards. Answer B describes a visual perceptual illusion known as the autokinetic effect, in which a stationary light in a dark room appears to move if stared at long enough. Answer D describes the Barnum effect, which is the tendency for a person to give high accuracy ratings to character descriptions that are supposedly tailored specifically to him or her but are in fact vague and generally applicable to most people.

209
Q

Type I Error is:
Select one:

A.
Rejecting the H0 when it should have been retained.

B.
Rejecting the H1 when it should have been retained.

C.
Failing to reject the H0 when it should have been rejected.

D.
Failing to reject the H1 when it should have been rejected.

A

The correct answer is A.

Rejecting the null hypothesis when it is true is referred to as Type I error. Failing to reject a false null hypothesis is referred to as Type II error (answer B). The H1 is not tested for rejection (answer D). If the H0 is rejected, then the H1 is accepted as the alternate to the rejected H0 (answer C).

210
Q

Which of the following is true of pyromania?
Select one:

A.
It is often due to impaired judgment.

B.
It is usually part of a conduct disorder.

C.
It is associated with indifference to the consequences to life or property caused by the fire.

D.
It can be diagnosed after a single act of deliberate fire-setting.

A

The correct answer is C.

The fire-setting is associated with indifference to its consequences. Answer A is incorrect because the fire-setting in pyromania is not due to impaired judgment (as in psychosis or intellectual disability). Answer B is incorrect because fire-setting as part of conduct disorder would be diagnosed as conduct disorder, not pyromania. Answer D is incorrect because the diagnosis includes the criteria of “purposeful fire setting on more than one occasion” (DSM-5), not just once.

211
Q

Tom works for a construction company that is organizing a group to work on a demolition project. Tom is selected to be part of a small, well-compensated group responsible for demolishing an old building in the middle of the Illinois countryside in a reasonable amount of time. Tom’s ideas and assigned tasks for the projects are attentively considered and appreciated. He is given honest feedback about his contribution throughout the work and feels appreciated and respected for what he is bringing to the group and the assignment. It is quite likely that:
Select one:

A.
Tom will consider adding an extra task to his current duties.

B.
Tom will not be engaging in social loafing.

C.
Tom will contribute to the group work by being highly motivated and provide great personal output.

D.
Tom will provide a great contribution to the team’s work after asking for the group’s size to be increased.

A

The correct answer is C.

Answer A is not likely to happen, as Tom will be satisfied with the level of work he is assigned, finding no need to add extra tasks to what looks like an optimal work condition. Answer B is a correct situation, but if you were asked a similar question on the EPPP and had to choose “the best answer,” this would not be it, as it would be incomplete compared to Answer C. To be logical and congruent with what is being asked does not necessarily make it the correct answer for the question. Answer D is definitely not likely, as based on the social loafing description, it is predicted that Tom will appreciate the smaller group size.

212
Q

With reference to the diathesis-stress model, which score is matched with the correct item?
Select one:

A.
Divorce is 50 points.

B.
Retirement is 45 points.

C.
Foreclosure is 73 points.

D.
Death of a close friend is 30 points.

A

The correct answer is B.

The score for divorce is 73 points, which makes answer A incorrect. The score for foreclosure of a home is 30 points, not 73 points, which makes Answer C incorrect. Death of a close friend is 37 points, not 30 points, which makes answer D incorrect.

213
Q

__________ would be most likely to conduct a study on learned optimism.
Select one:

A.
Seligman

B.
Watson

C.
Beck

D.
Maslow

A

The correct answer is A.

Martin Seligman (A) is best known for his work in attribution style. He focused both on pessimistic attributions for negative events that are common in depression (internal, stable, and global) and attributions of negative events of an optimistic individual (external, unstable, and specific). Watson (B) is best known for his behavioral work. Beck (C) is best known for his Cognitive therapy. And Maslow (D) is best known for his hierarchy of needs.

214
Q

For complex tasks in smaller groups, __________ is to centralized communication networks as __________ is to decentralized communication networks.
Select one:

A.
higher satisfaction and poorer performance; lower satisfaction and better performance

B.
lower satisfaction and poorer performance; higher satisfaction and better performance

C.
higher satisfaction and better performance; lower satisfaction and poorer performance

D.
lower satisfaction and poorer performance; higher satisfaction and poorer performance

A

The correct answer is B.

In centralized communication networks in smaller groups (four to six), not all group members have access to one another to receive information. Instead, information is passed from one key person to others in the group. Sometimes, such networks are pictured as wheels, with peripheral members able to communicate with each other only by going through the central “spoke” of the wheel. In the workplace, centralized communication networks are associated with low job satisfaction, low speed on complex tasks, and low accuracy on complex tasks. This type of network, however, allows for the minimal amount of communication needed for high speed and accuracy on simple tasks. While the overall satisfaction level is low, the person in the center, the spoke, usually reports high levels of satisfaction, especially when the tasks are simple. In contrast, decentralized communication networks in smaller groups allow more, or sometimes all, group members to be involved in the communication. Such networks are often pictured as circles, or as completely connected systems in which all members are linked to all other members. Decentralization is associated with higher levels of satisfaction but low speed and accuracy on simple tasks. Decentralization allows for efficient and effective communication, which increases speed and accuracy on complex tasks.

215
Q

Which of the following structures is NOT located in the midbrain?
Select one:

A.
Inferior colliculus

B.
Red nucleus

C.
Mammillary bodies

D.
Ventral tegmental area

A

The correct answer is C.

The inferior colliculus (answer A), red nucleus (answer B), and tegmentum (answer D) are all structures in the midbrain, also known as the mesencephalon. The mammillary bodies are structures in the limbic system and are a part of the hypothalamus, which is located in the diencephalon region of the brain (the area right above the mesencephalon).

216
Q

Identifying alternative behaviors to replace undesirable behavior is an integral task in:
Select one:

A.
Contingency management.

B.
Behavioral diagnosis.

C.
Implosion therapy.

D.
Differential reinforcement of other behavior.

A

The correct answer is A.

Contingency management is an operant conditioning technique focused on replacing undesirable behaviors with desirable behaviors. To accomplish this task, reinforcement is delivered when the person engages in desirable, rather than undesirable, behaviors. The process involves identifying target behaviors to reduce, identifying target behaviors to increase, identifying reinforcement schedules, identifying the source of the provision of reinforcement, and identifying the content of the reinforcement. Behavioral diagnosis (answer B) refers to diagnostic categories that have a behavioral component (e.g., oppositional defiant disorder). Implosion therapy (answer C), also known as flooding, is a classical conditioning technique focused on extinguishing anxious responding to a feared stimulus. Differential reinforcement of other behavior (answer D) involves the process of reinforcing all other behaviors to reduce the likelihood of an undesirable behavior.

217
Q

Which of the following diagnoses would most likely lead to a determination of insanity in a court of law?
Select one:

A.
Borderline personality disorder

B.
Asperger’s disorder

C.
Major depressive disorder

D.
Schizophrenia, paranoid type

A

The correct answer is D.

Individuals with schizophrenia are most likely to be impaired with such severity that they are unable to distinguish right from wrong and understand the consequences of their behavior. Psychotic disorders that involve a loss of reality testing are most likely to be the basis of an NGRI plea; individuals with the other three disorders do not typically have a loss of reality testing, which makes answers A, B, and C incorrect.

218
Q

The main difference between hierarchical regression and stepwise regression is that:
Select one:

A.
F-tests are only used in hierarchical regression and not in stepwise regression.

B.
the researcher determines the order of variable entry in hierarchical, whereas it is statistically determined or data-driven in stepwise regression.

C.
dummy variables can only be used in stepwise regression.

D.
hierarchical regression is affected by multicollinearity, whereas stepwise is not.

A

The correct answer is B.

With a hierarchical regression, the researcher adds independent variables in an ordered series of steps; in a stepwise regression, the process of adding and removing predictors from the regression is driven by the data themselves.

219
Q

Covert sensitization is a __________ technique used to decrease undesirable behavior using negative imagery.
Select one:

A.
behavior therapy

B.
cognitive therapy

C.
Gestalt therapy

D.
rational emotive therapy

A

The correct answer is A.

As the goal of covert sensitization is to decrease undesirable behavior, this should serve as a cue to behavior therapy as the correct answer. Further, covert sensitization uses aversive mental consequences, which is a behavioral technique used to change undesirable behaviors. Cognitive therapy (answer B) may change behavior, but it focuses on the process of changing thoughts directly, which may have an impact on emotional and/or behavioral functioning. Rational emotive therapy (answer D) is a cognitive therapy that challenges irrational beliefs. Gestalt therapy (answer C) is a therapy focused on the reunification of the separated parts of the self into a coherent whole.

220
Q

According to HIPAA, Dr. Jones must keep his clients’ information on file for:
Select one:

A.
10 years.

B.
7 years.

C.
8 years.

D.
6 years.

A

The correct answer is D.

Records must be kept for six years. Answers A, B, and C refer to incorrect periods of time.

221
Q

Joe has a tumor that disrupted his right medial optic nerve. Where is he likely to experience a loss of vision?
Select one:

A.
The right visual field of his left eye

B.
The right visual field of his right eye

C.
The left visual field in both eyes

D.
The left visual field of his right eye

A

The correct answer is B.

The medial optic nerve from the right eye sends information from the right visual field in the right eye to the brain, whereas the medial optic nerve in the left eye sends information from the left visual field in the left eye. Therefore, damage to the right medial optic nerve will cause blindness in the right visual field in the right eye. Information from both medial optic nerves crosses to the contralateral (opposite) hemisphere via the optic nerve.

222
Q

Chronic pain is best treated with which of the following medications?
Select one:

A.
Hydrocodone

B.
Venlafaxine

C.
Valium

D.
Lithium

A

The correct answer is B.

Venlafaxine, also known as Effexor, is a serotonin and norepinephrine reuptake inhibitor (SNRI). SNRIs are effective in treating chronic pain, are more effective than SSRIs, and have fewer side effects than tricyclic antidepressants. Hydrocodone is an opiate that is used in the treatment of moderate acute pain. Because it is highly addictive, it is not used for the treatment of chronic pain. Valium is a benzodiazepine that is also highly addictive. It is used to treat sleep disorders, anxiety, and panic attacks. Lithium is a mood stabilizer used to treat mania and aggression.

223
Q

Abby, a 29-year-old woman, comes to therapy after the dissolution of a romantic relationship. She tells the therapist that she was referred by her primary care physician, whom she refers to as “my dear, dear friend Martin.” Abby is effusive and theatrical while describing her feelings about the ending of her romance, but provides few details about the relationship itself. Throughout the session, Abby emphasizes her remarks by leaning forward to place her hand just above the therapist’s knee. Based on this information, the most appropriate diagnosis for Abby would be:
Select one:

A.
Borderline personality disorder.

B.
Narcissistic personality disorder.

C.
Histrionic personality disorder.

D.
Dependent personality disorder.

A

The correct answer is C.

Abby displays the dramatic, superficial, and overly emotional qualities characteristic of histrionic personality disorder. Answer A is incorrect, as she does not have the intense affective lability of borderline personality disorder. Answer B is also incorrect, as Abby does not portray the entitlement of narcissistic personality disorder. Answer D is incorrect, as she does not show the global dependency of dependent personality disorder.

224
Q

A(n) __________ focuses on equalizing power differentials and social system changes rather than individual change.
Select one:

A.
advocacy consultant

B.
behavioral consultant

C.
mental health advocate

D.
feminist therapist

A

The correct answer is A.

Advocacy consultants focus on equalizing power differentials and focus on social systems rather than individual change. Behavioral consultants (answer B) focus on individual behavior changes. Mental health advocates (answer C) focus on assisting people in involuntary commitment hearings. Feminist therapists (answer D) do take a strong position on the social structures that affect individuals and work on the power differential inherent in the therapeutic relationship, but undertake their work primarily on an individual basis.

225
Q

When feeling slighted by her boyfriend, Meghan is excessively callous toward him, which was how she coped with similar situations with her father. Meghan is exhibiting which interpersonal phenomenon?
Select one:

A.
Projection

B.
Parataxic distortions

C.
Projective identification

D.
Displacement

A

The correct answer is B.

In the vignette, it is important to notice the wording, which demonstrates a recapitulation of prior interpersonal coping skills in the current interpersonal relationship. This process is explained in the interpersonal theory and psychotherapy model. Of the options above, only parataxic distortions (answer B) are a component of IPT. Furthermore, transference-like experiences are argued to be a result of parataxic distortions. Answers A, C, and D are defense mechanisms that are not included in the IPT model.